You are on page 1of 44

Practice Test 6:

Answers and
Explanations
SAT Premium Prep

PRACTICE TEST 6 ANSWER KEY

Section 1: Section 2: Section 3: Section 4:


Reading Writing & Language Math (No Calculator) Math (Calculator)
1. B 27. B 1. C 23. A 1. B 11. B 1. A 20. D
2. D 28. C 2. B 24. B 2. B 12. A 2. A 21. D
3. A 29. B 3. D 25. D 3. D 13. A 3. C 22. D
4. B 30. C 4. B 26. C 4. A 14. C 4. C 23. B
5. A 31. A 5. D 27. B 5. C 15. C 5. B 24. A
6. D 32. A 6. D 28. B 6. D 16. 8/10, 6. C 25. B
7. A 33. B 7. C 29. A 7. C 4/5, 7. D 26. B
8. B 34. A 8. C 30. D 8. B or 8. D 27. C
9. D 35. D 9. D 31. B 9. D 0.8 9. C 28. C
10. C 36. C 10. D 32. C 10. B 17. 30 10. D 29. A
11. A 37. C 11. B 33. A 18. 12 11. B 30. A
12. B 38. A 12. D 34. D 19. 2 12. D 31. 5/9,
13. B 39. B 13. B 35. A 20. 5 13. A .555,
14. C 40. D 14. C 36. C 14. B or
15. A 41. C 15. A 37. C 15. C .556
16. B 42. D 16. D 38. A 16. A 32. 430
17. D 43. C 17. D 39. B 17. C 33. 1/3
18. D 44. C 18. C 40. A 18. D or
19. B 45. A 19. A 41. C 19. A .333
20. D 46. D 20. A 42. B 34. 672
21. A 47. B 21. B 43. B 35. 5, 6,
22. B 48. C 22. C 44. C or 7
23. D 49. D 36. 180
24. A 50. B 37. 28
25. D 51. D 38. 1.18
26. C 52. B

2   |   Practice Test 6: Answers and Explanations


SAT Premium Prep

PRACTICE TEST 6 ANSWERS AND EXPLANATIONS

Section 1: Reading
1. B The question asks how the narrator’s attitude shifts over the course of the passage. This is the
first question in a paired set, so it can be done in tandem with Q2. Look at the answer choices
for Q2 first. The lines for (2A) describe the moments before the ship struck upon the sand. Since
this description does not address the narrator’s attitude, eliminate (2A). The lines for (2B) state
that a wave...came upon [the narrator] again and submerged him in the water. Like (2A), (2B)
does not address the narrator’s attitude. Eliminate (2B). In the lines for (2C), the narrator says,
I resolved to hold fast by a piece of rock, and so to hold my breath, if possible, till the wave went back.
Being resolved does describe the narrator’s attitude, but there is no shift, so eliminate (2C).
In the lines for (2D), the narrator says, it is impossible to express, to the life, what the ecstasies
and transports of the soul are, when it is so saved…out of the very grave. These lines capture the
shift in the narrator’s attitude: when the shipwreck occurred, he says, we expected we should
all have perished immediately (lines 7–8). In lines 40–43, the ecstasies and transports of the soul
describe his feelings when he realizes he has survived. These lines support (1B), so draw a line
connecting (2D) and (1B). Without any support in the answer choices from Q2, (1A), (1C),
and (1D) can be eliminated. The correct answers are (1B) and (2D).

2. D See explanation for Q1.

3. A The question asks what the phrase broke over her most nearly means in line 6. Go back to the
text, find the phrase broke over her, and cross it out. Then read the window carefully, using
context clues to determine another phrase that would fit in the text. The text says that the sea
broke over [the ship] in such a manner that we expected we should all have perished immediately.
Therefore, broke over her could be replaced by a phrase such as “slammed into the ship.” Elimi-
nate answer choices that don’t match this prediction. Choice (A) matches the prediction, so
keep it. Choice (B) goes too far, as there is no evidence in the text that the sea destroyed the ship.
Eliminate (B). Eliminate (C), as the ship had already hit the sand in the moment before the
wave slammed into it. Like (B), (D) goes beyond the scope of the text, as there is no evidence
that the wave fragmented the ship’s frame. Eliminate (D). The correct answer is (A).

4. B The question asks for the purpose of the sentence in lines 34–40. Use the given line reference
to find the window. In lines 34–40 the narrator states, to my great comfort, I clambered up the
clifts and sat upon the grass, free from danger. The narrator goes on to say, I was now landed and
safe on shore, and began to look up and thank God my life was saved. Eliminate answer choices
that don’t match this prediction. Choice (A) does not match the prediction, as the narrator
never states the sea was stronger than [he] anticipated, so eliminate (A). Choice (B) matches
the prediction, so keep it. While (C) matches with the prediction when the narrator states he
was now landed and safe on shore, this is not the main purpose of the sentence. Eliminate (C).

Practice Test 6: Answers and Explanations  |   3


SAT Premium Prep

Choice (D) is a Right Words, Wrong Meaning trap answer, as the religious side of the narrator
calls back to thanking God, but there is no evidence that this is new or not previously seen.
Eliminate (D). The correct answer is (B).

5. A The question asks what is implied about the narrator by the reference in lines 43–50. Use the
given line reference to find the window. In lines 43–50, the narrator suggests that he under-
stands why a malefactor, who has the halter about his neck, is tied up…and has a reprieve brought
to him, may be overwhelmed by such a surprise, as sudden joys, like griefs, confound at first.
Eliminate answer choices that don’t match this prediction. Choice (A) matches this prediction,
so keep it. Choice (B) is a Right Words, Wrong Meaning trap answer: it references a surgeon
who lets a patient bleed, but the narrator uses this image metaphorically. Eliminate (B). Choice
(C) is also a Right Words, Wrong Meaning trap answer: the narrator never indicates that
he literally had a noose around his neck. Eliminate (C). Eliminate (D) because the narrator’s
reference is about understanding what it is like to be suddenly saved, not about understanding
what it’s like to be a criminal. Eliminate (D). The correct answer is (A).

6. D The question asks what drive most nearly means in line 49. Go back to the text, find the
word drive, and cross it out. Then read the window carefully, using context clues to determine
another word that would fit in the text. The text says that a surgeon bleeds the prisoner at the
moment of reprieve so that the surprise may not drive the animal spirits from the heart. Therefore,
drive could be replaced by a word such as “force.” Eliminate answer choices that don’t match
this prediction. Cause does not match “force,” so eliminate (A). Accelerate does not match
“force,” so eliminate (B). Imply does not match “force,” so eliminate (C). Expel matches “force,”
so keep (D). The correct answer is (D).

7. A The question asks what the phrase contemplation of my deliverance most nearly means in line
53. Go back to the text, find the phrase contemplation of my deliverance, and cross it out. Then
read the window carefully, using context clues to determine another phrase that would fit in
the text. The narrator describes himself as wrapped up in a contemplation of my deliverance and
reflecting upon all my comrades that were drowned, and that there should not be one soul saved
but myself. Eliminate answer choices that don’t match this prediction. Choice (A) matches the
prediction, so keep it. The narrator does not exhibit any foresight, so eliminate (B). Choice (C)
can be eliminated as the narrator has already been saved. Eliminate (D) because while the
narrator may be an exemption in that he was the only one saved from the shipwreck, he has no
understanding of why he was saved. The correct answer is (A).

8. B The question asks what the narrator indicates about the other shipmates on the voyage. This is
the first question in a paired set, but it is easy to find, so it can be done on its own. Since there
is no line reference, use the order of the questions to find the window. Q7 asks about line
53, so scan the passage beginning with line 54, looking for information about the narrator’s
shipmates. In lines 55–56, the narrator reflects on all my comrades that were drowned, and that
there should not be one soul saved but myself. Eliminate answer choices that don’t match this

4   |   Practice Test 6: Answers and Explanations


SAT Premium Prep

prediction. Eliminate (A) as there is no indication that the shipmates had different motivations
for the journey than the narrator did. Choice (B) matches the prediction, so keep it. Eliminate
(C) as none of the shipmates survived or discovered land and found shelter. Eliminate (D) as the
passage never states the shipmates failed to determine whereabout in the world they were. The
correct answer is (B).

9. D The question is the best evidence question in a paired set. Because the previous question was
easy to find, simply look at the lines used to answer Q8. Lines 55–56 provided the prediction
for Q8: all my comrades that were drowned, and that there should not be one soul saved but myself.
Eliminate (A), (B), and (C). The correct answer is (D).

10. C The question asks about the narrator’s view of his struggle to safety. Since this is a general ques-
tion, it should be answered after the specific questions. In lines 5–9 the narrator states, the ship
struck upon the sand, and in a moment, her motion being so stopped, the sea broke over her in such
a manner that we expected we should all have perished immediately. The narrator also states in
lines 37–39, I was now landed and safe on shore, and began to look up and thank God that my life
was saved. Eliminate answer choices that don’t match this prediction. There is nothing prom-
ising about the narrator’s struggle—in fact, the narrator says he expected to have perished— so
eliminate (A). There is nothing enjoyable about his experience of the shipwreck, so eliminate
(B). Harrowing but relieving matches the prediction, so keep (C). The narrator never indicates
that he was intrigued by being shipwrecked, so eliminate (D). The correct answer is (C).

11. A The question asks which type of industry would be the most difficult to relocate. Since there is no
line reference, use lead words and the order of the questions to find the window. Since this is
the first question, start skimming at the beginning of the passage to find the part of the passage
that discusses relocation of industries. Lines 34–39 state, A textile factory is a stand-alone
entity that can be put pretty much anywhere in the world where labor is abundant. By contrast, a
biotech lab is harder to export, because you would have to move not just one company but an entire
ecosystem. Therefore, a biotech company would be hardest to relocate. Eliminate answer choices
that don’t match this prediction. Choice (A) matches this prediction, so keep it. Choice (B) is
a Right Answer, Wrong Question: the passage indicates that a textile factory would be easier to
relocate than a biotech lab. Eliminate (B). Neither (C) nor (D) is mentioned in the window for
this question, so eliminate both (C) and (D). The correct answer is (A).

12. B The question asks for the best summary of the third paragraph. Use the given paragraph refer-
ence to find the window. According to lines 39–45, cities are complex, interrelated environments
that foster the generation of new ideas and social interactions among workers tend to generate
learning opportunities. The paragraph goes on to say that once a city attracts innovative workers
and innovative companies, its economy changes in ways that make it even more attractive to other
innovators. Eliminate answer choices that don’t match this prediction. Choice (A) is a Mostly
Right, Slightly Wrong trap answer: the paragraph suggests that social interaction can spark
innovation but does not say that social interaction is a requirement for a company’s success.

Practice Test 6: Answers and Explanations  |   5


SAT Premium Prep

The word must is too strong to match the passage, so eliminate (A). Keep (B) because it matches
the prediction. Choice (C) is a Mostly Right, Slightly Wrong trap answer: the paragraph states
that a city may grow because it attracts smart people and innovative businesses but does not
indicate this is the only way a city can grow. Eliminate (C). There is no information on cell
phone or internet access in the passage, so eliminate (D). The correct answer is (B).

13. B The question asks what the word foster most nearly means in line 41. Go back to the text,
find the word foster, and cross it out. Then read the window carefully, using context clues to
determine another word that would fit in the text. The text says that cities are environments
that foster the generation of new ideas and new ways of doing business. Therefore, foster could be
replaced by a phrase such as “bring about.” Eliminate answer choices that don’t match this
prediction. Adopt does not match “bring about,” so eliminate (A). Keep (B) because encourage
matches “bring about.” Neither shelter nor entertain matches “bring about,” so eliminate (C)
and (D). Note that (A) and (C) are Could Be True trap answers based on other meanings of
foster that are not supported by the text. The correct answer is (B).

14. C The question asks for the best description of the economic development of some developing coun-
tries. This is the first question in a paired set, so it can be done in tandem with Q15. Look
at the answer choices for Q15 first. The lines for (15A) say that the most important economic
development of the past decade is the incredible improvement in the standard of living in devel-
oping nations such as Brazil, China, Poland, Turkey, India, and even some African countries.
This supports the phrases reduced disparity and improved standard of living in (14C). Draw
a line connecting (15A) and (14C). The lines for (15B) reference the fact that inequality has
plummeted...at a global level. This answer doesn’t support any of the answer choices in Q14, so
eliminate (15B). The lines for (15C) reference the catch-up experienced by the American South,
which does not address the question about developing countries. Eliminate (15C). The lines for
(15D) state, Many southern states were significantly poorer than the rest of the country in the 1960s
but grew more rapidly in the following decades. This information does not address Q14. Elimi-
nate (15D). Without any support in the answers from Q15, (14A), (14B), and (14D) can be
eliminated. The correct answers are (14C) and (15A).

15. A See explanation for Q14.

16. B The question asks what the word uneven most nearly means in line 78. Go back to the text,
find the word uneven and cross it out. Then read the window carefully, using context clues
to determine another word that would fit in the text. The text says, the process of catching
up was geographically uneven. Some southern cities...grew much faster than others. Therefore,
uneven could be replaced by a phrase such as “not the same.” Eliminate answer choices that
don’t match this prediction. Rugged doesn’t match “not the same,” so eliminate (A). Unequal
matches “not the same,” so keep (B). Neither intermittent nor bumpy matches “not the same,”,
so eliminate (C) and (D). Note that (A), (C), and (D) are Could Be True trap answers based
on other meanings of uneven that are not supported by the text. The correct answer is (B).

6   |   Practice Test 6: Answers and Explanations


SAT Premium Prep

17. D The question asks what the passage implies about Shanghai, China. This is the first question
in a paired set, but it is easy to find, so it can be done on its own. Since there is no line refer-
ence, use lead words and the order of the questions to find the window. Q16 asks about line
78, so start at line 78 and skim for the lead word Shanghai. Lines 83–89 state, Shanghai has
reached a per capita income close to that of a rich nation. Its students outperform American and
European students…Its public infrastructure is better than that of most American cities. But agri-
cultural communities in western China have made much less progress. Eliminate answer choices
that don’t match this prediction. Choice (A) is a Right Words, Wrong Meaning trap answer:
while the students in Shanghai outperform American and European students in standardized
tests, the passage does not indicate that students in Shanghai are the smartest in the country.
Eliminate (A). Eliminate (B) because the passage states that agricultural communities in western
China have made much less progress than Shanghai. Therefore, Shanghai is not representative of
China as a whole. Choice (C) is a Mostly Right, Slightly Wrong trap answer: the passage points
to an income disparity between Shanghai and agricultural communities in western China.
However, the passage does not imply that such an income disparity exists within Shanghai
itself. Eliminate (C). Keep (D) because it matches the prediction: Shanghai is making greater
advances than agricultural communities in western China. The correct answer is (D).

18. D The question is the best evidence question in a paired set. Because the previous question was
easy to find, simply look at the lines used to answer Q17. Lines 83–89 provided the prediction
for Q17. Eliminate (A) and (B). Lines 83–84 state that Shanghai has reached a per capita income
close to that of a rich nation but don’t include information about other regions in China. Lines
88–89 include the contrast between Shanghai and agricultural communities in western China.
Eliminate (C) and keep (D). The correct answer is (D).

19. B The question refers to figure 1 and asks how to characterize the years 1993–1997. Locate the
years 1993–1997 in figure 1. While the graph shows an overall decline in manufacturing
jobs, the years 1993–1997 show a brief increase in those jobs followed by a decline. Choice
(A) is inconsistent with the information in the graph. The graph shows that for a short time
during those years, there was an increase in manufacturing jobs. Eliminate (A). Choice (B)
is consistent with the graph. In all other years, manufacturing jobs are declining, while the
period of 1993–1997 saw a brief increase in the number of manufacturing jobs. Choice (C) is
inconsistent with the information presented in the graph. The years 1993–1997 show a brief
increase in the number of manufacturing jobs. Since the years after that period show a decline
in manufacturing jobs, this was not a permanent reversal. Eliminate (C). The y-axis of figure
1 is in millions of jobs, and therefore even a small increase could be a significant change in the
number of jobs. For this reason, eliminate (D). The correct answer is (B).

Practice Test 6: Answers and Explanations  |   7


SAT Premium Prep

20. D The question asks for the author’s most likely view of the information in figure 2. Carefully read
figure 2, then consider how it relates to the passage. Figure 2 provides a list of U.S. cities with
their geographical regions. It shows each city’s median household income in 2010 and in 2019. It
also shows the percent change in each city’s income between 2010 and 2019. In the passage, the
author describes the possibility that the good jobs, now concentrated in high-cost locations such
as Silicon Valley and Boston, will quickly disperse to low-cost locations. He provides examples of
the income disparity between locations and says that in this version of the future…Cities that
have been lagging behind—the Clevelands, the Topekas, and the Mobiles—will grow much faster,
while San Francisco, New York, Seattle, and similar cities will decline. Then he says that the data
don’t support this view. In fact, the opposite has been happening. In other words, income levels in
cities such as San Francisco, New York, and Seattle have been rising quickly, while income levels
in cities such as Cleveland, Topeka, and Mobile are not growing as quickly. In the final para-
graph, he also states, Some southern cities—Austin, Atlanta, Durham, Washington, D.C., Dallas,
and Houston, for example—grew much faster than others, thus increasing the disparity among
communities in the South. Work through the answer choices, considering both the passage and
the figure. Choice (A) states that the information in the figure is inconsistent with information
discussed in the passage, while (D) states that the information is evidence of a continuing trend
discussed in the passage. Figure 2 shows the highest percent growth in income in Seattle and San
Francisco and the lowest percent growth in income in Cleveland and Topeka. Comparing thee
two Southern cities in the figure, the income growth in Austin is higher than that in Mobile. In
both cases, the data in the table is consistent with the trends the author discusses. Eliminate
(A) and keep (D). Choice (B) states that the information in the figure is potentially relevant but
too general to be useful. The figure provides income data for specific cities in specific years, and
there is no indication in the passage that the author considers this type of data to be too general.
Eliminate (B). Choice (C) states that the information in the figure reflects the author’s own
experiences, but the figure is about median income growth in eight different cities; it doesn’t
reflect one person’s experiences. Eliminate (C). The correct answer is (D).

21. A The question asks what figure 2 shows about median household income between 2010 and 2019.
Work through each answer choice using the figure. Figure 2 shows that San Francisco had the
highest median household income in 2010, and San Francisco also had the greatest percent growth
in income. Keep (A). Figure 2 doesn’t include any information about which cities were consid-
ered the most desirable place to live, so eliminate (B). Mobile, Alabama, a city in the Southern
region, had a growth rate of only 25%, while the two cities in the Northeastern region had
growth rates of 42.4% and 58.4% respectively. Eliminate (C). There is no information about
population size in figure 2, so eliminate (D). The correct answer is (A).

22. B The question asks for the primary purpose of the passage. Because this is a general question,
it should be done after the specific questions. The passage describes how a group of scien-
tists identified the gene responsible for a mysteriously (and dangerously) sweatless family by
mapping its genomes. That research, which was used to help people who sweat too much,

8   |   Practice Test 6: Answers and Explanations


SAT Premium Prep

emphasizes the role of sweat in human evolution. Eliminate answer choices that don’t match
this prediction. Eliminate (A) because only the first paragraph discusses the family’s struggles;
most of the passage is about the science behind the family’s disorder. Keep (B) because it
matches the prediction. Eliminate (C) because the passage does not evaluate new technologies.
Eliminate (D) because the family’s disorder was not a previously-solved problem. The correct
answer is (B).

23. D The question asks for the author’s attitude toward genome mapping. Since there is no line
reference, use lead words and the order of the questions to find the window. Since this is
the first specific question, scan the passage from the beginning, looking for the lead words
genome mapping. Lines 11–14 state that by mapping the genome of each individual in this family,
researchers from Uppsala University in Sweden identified a single genetic mutation responsible
for the condition. The genome mapping solved the mystery of the family’s condition, which
is a positive outcome. The author’s attitude to genome mapping is positive. Eliminate answer
choices that don’t match this prediction. Eliminate (A) because disapproval is a negative atti-
tude. Eliminate (B) because concern is also negative. Eliminate (C) because elation means
“great happiness,” which is too strong. Keep (D) because it matches the prediction. The correct
answer is (D).

24. A The question asks what the phrase decoding the mystery suggests. Use the given line reference
to find the window. Lines 10–14 indicate that scientists are decoding the mystery behind the
disorder and that scientists identified a single genetic mutation responsible for the condition. Elim-
inate answer choices that don’t match this prediction. Keep (A) because it matches the predic-
tion. Choice (B) is a Mostly Right, Slightly Wrong trap answer: lines 12–13 do say that the
researchers are mapping the genome of each individual in this family, but the passage does not say
that they are close to finishing the genome map. Eliminate (B). Choice (C) is a Right Answer,
Wrong Question trap answer: the passage does say that the children stay in the cellar, but
lines 7–8 give the reason: because in daylight, they’ d overheat and go unconscious. Eliminate (C).
Choice (D) is a Mostly Right, Slightly Wrong trap answer: although the passage does mention
Mendelian disorders, the word only is too strong to be supported by the passage. Eliminate (D).
The correct answer is (A).

25. D The question asks what the word bizarre most nearly means in line 11. Go back to the text,
find the word bizarre, and cross it out. Then read the window carefully, using context clues
to determine another word that would fit in the text. The text says, Now a group of scientists
are decoding the mystery surrounding this bizarre disorder. In the first paragraph, the disorder is
described as unique and rare. Therefore, bizarre could be replaced by a word such as “rare.”
Eliminate answer choices that don’t match this prediction. Eccentric and offbeat mean
“unconventional,” which does not match “rare,” so eliminate (A) and (C). Ludicrous means
“ridiculous,” which does not match “rare,” so eliminate (B). Unusual matches “rare,” so keep
(D). Note that (A), (B), and (C) are Could Be True trap answers based on other meanings of
bizarre that are not supported by the text. The correct answer is (D).

Practice Test 6: Answers and Explanations  |   9


SAT Premium Prep

26. C The question asks what the author suggests about anhidrosis. This is the first question in a paired
set, but it is easy to find, so it can be done on its own. Since there is no line reference, use lead
words and the order of the questions to find the window. Q25 asks about line 11, so scan the
passage beginning with line 12, looking for the lead word anhidrosis. Starting at line 20, the
passage says, Dahl notes that anhidrosis has been seen before, but usually in conjunction with other
skin defects. Eliminate answer choices that don’t match this prediction. Choice (A) is a Right
Answer, Wrong Question trap answer: excessive sweating refers to hyperhidrosis, not anhidrosis.
The passage says that people with anhidrosis cannot sweat (line 3). Eliminate (A). Choice (B) is
a Right Words, Wrong Meaning trap answer: lines 6–8 say, In order to remain cool, the kids
spend their days in their house’s cellar, because in daylight, they’ d overheat and go unconscious, so
anhidrosis does not cause people to feel too cool. Eliminate (B). Keep (C) because it matches
the prediction. Choice (D) is a Right Answer, Wrong Question trap answer: the passage states
that two percent of the population suffers from hyperhidrosis, not anhidrosis. Eliminate (D). The
correct answer is (C).

27. B The question is the best evidence question in a paired set. Because the previous question was
easy to find, simply look at the lines used to answer Q26. Lines 20–22 provided the prediction
for Q26: Dahl notes that anhidrosis has been seen before, but usually in conjunction with other
skin defects. Eliminate (A), (C), and (D). The correct answer is (B).

28. C The question asks what is true of ITPR2. This is the first question in a paired set, so it can be
done in tandem with Q29. Look at the answer choices for Q29 first. The lines for (29A) say,
Now a group of scientists are decoding the mystery surrounding this bizarre disorder. These lines do
not address Q28, so eliminate (29A). The lines for (29B) say that ITPR2 is the gene responsible
for controlling sweat production, and knocking it out can stop sweat secretion altogether. These
lines support (28C) since changes to the gene can inhibit sweat production, which is a physical
disorder. Draw a line connecting (28C) with (29B). The lines for (29C) mention researcher
Niklas Dahl and his quest for understanding single-gene diseases. The lines do not address Q28,
so eliminate (29C). The lines for (29D) discuss the protein IP3R2, which forms a calcium
channel in the brain that releases calcium when opened. At first glance, this information may
seem to match (28B), but these lines are about what the protein IP3R2 does, not about what
is true of the gene ITPR2. Eliminate (29D). Without any support in the answers from Q29,
(28A), (28B), and (28D) can be eliminated. The correct answers are (28C) and (29B).

29. B See explanation for Q28.

30. C The question asks what the author suggests about Botox. Use the given line reference to find the
window. Lines 49–52 say, Botox has proven to be somewhat effective against hyperhidrosis, but the
treatments can be painful and awkward (getting injections in your armpits cannot be fun). Elimi-
nate answer choices that don’t match this prediction. Choice (A) is a Right Answer, Wrong
Question trap answer: the passage states that lowering IP3R2 levels inhibits production of the
calcium channel. The passage doesn’t state that Botox has this effect, so eliminate (A). Elimi-

1 0   |   Practice Test 6: Answers and Explanations


SAT Premium Prep

nate (B), which is another Right Answer, Wrong Question trap answer: the passage states that
lowering IP3R2 levels can reduce excessive sweating by 60%. Keep (C) because it matches the
prediction. Eliminate (D) because pungent skin secretions are not discussed in the window. The
correct answer is (C).

31. A The question asks about the likely purpose of the parenthetical information. Use the given line
reference to find the window. Lines 49–52 say, Botox has proven to be somewhat effective against
hyperhidrosis, but the treatments can be painful and awkward (getting injections in your armpits
cannot be fun). The parenthetical information explains why Botox treatments are painful and
awkward. Eliminate answer choices that don’t match this prediction. Keep (A) because it
matches the prediction. Eliminate (B) because the parenthetical information does not contain
a definition. Eliminate (C) because there is no conclusion in the parentheses. Eliminate (D)
because the parenthetical information does not resolve a discrepancy. The correct answer is (A).

32. A The question asks how, according to Hamilton, political appointments made by a single indi-
vidual differ from those made by a group. Since this is a general question, it should be answered
after the specific questions. In lines 23–24, the passage states that a single individual will have
fewer personal attachments to gratify, than a body of men. In lines 27–31, the passage also states
that A single well-directed man, by a single understanding, cannot be distracted or warped by that
diversity of views, feelings, and interests, which frequently distract and warp the resolutions of a
collective body. Therefore, Hamilton believes that a single individual is less likely to be biased or
swayed by multiple wants and desires. Eliminate answer choices that don’t match this predic-
tion. Keep (A) because it matches the prediction. Eliminate (B) because neither power nor
esteem is mentioned as something that the single individual has but the group does not. Choice
(C) is a Right Words, Wrong Meaning trap answer: lines 16–18 suggest that one man will
have a livelier sense of duty and a more exact regard to reputation; the passage does not say that
the political appointments will differ in their liveliness and exactness. Eliminate (C). Choice (D)
is a Mostly Right, Slightly Wrong trap answer: it partially matches the prediction in that the
single individual who cannot be distracted and warped by that diversity of views, feelings, and
interests could potentially be more efficient than a group at making political appointments;
however, convenience is not mentioned in the passage. Eliminate (D). The correct answer is (A).

33. B The question asks what the word peculiar most nearly means in line 13. Go back to the text,
find the word peculiar, and cross it out. Then read the window carefully, using context clues to
determine another word that would fit in the text. The text says that one man of discernment
is better fitted to analyze and estimate the peculiar qualities adapted to particular offices. There-
fore, peculiar could be replaced by a word such as “particular” or “specific.” Eliminate answer
choices that don’t match this prediction. Conventional does not match “particular,” so elimi-
nate (A). Distinctive matches “particular,” so keep (B). Neither eccentric nor wonderful matches
“particular,” so eliminate (C) and (D). Note that (C) is a Could Be True trap answer based on
another meaning of peculiar that is not supported by the text. The correct answer is (B).

Practice Test 6: Answers and Explanations  |   11


SAT Premium Prep

34. A The question asks what the word stations most nearly refers to in line 21. Go back to the text,
find the word stations, and cross it out. Then read the window carefully, using context clues
to determine another word that would fit in the text. The text says, He will, on this account,
feel himself under stronger obligations, and more interested to investigate with care the qualities
requisite the stations to be filled. Therefore, stations could be replaced by a word such as “posi-
tions.” Eliminate answer choices that don’t match this prediction. Keep (A) because it matches
the prediction. Municipal headquarters, transfer locations, and local buildings are physical loca-
tions and do not match the prediction of “positions” to be filled, so eliminate (B), (C), and
(D). Note that (B), (C), and (D) are Could Be True trap answers based on other meanings of
stations that are not supported by the text. The correct answer is (A).

35. D The question asks how Lee views the president’s ability to appoint qualified officials. Since there is
no line reference and no other questions about Passage 2 to use as a guide for chronology, work
backwards from the answer choices. Eliminate (A) because Lee does not suggest that the presi-
dent is unfairly biased. The passage does not indicate whether the president’s ability to appoint
qualified officials is more or less desirable than a general election, so eliminate (B). Eliminate (C)
because line 41 states, Officers may be appointed by the president alone, which indicates that the
president has been granted the power to appoint officials. Therefore, Lee doesn’t consider this
action to be an overreach of power. Keep (D) because lines 57–67 state that the exercise of any
considerable branch of power ought to be under some checks and controls and that the president’s
power may always be increased or decreased by the legislature…always keeping him, by the constitu-
tion, within certain bounds. The correct answer is (D).

36. C The question asks how Lee would most likely respond to a statement made by Hamilton. First
read the statement by Hamilton in lines 11–15: his view is that that one man of discernment is
better fitted to analyze and estimate the peculiar qualities adapted to particular offices, than a body
of men of equal or perhaps even of superior discernment. Q36 is the first question in a paired set,
so it can be done in tandem with Q37. Look at the answer choices for Q37. The lines for (37A)
refer to the fact that the president is permitted to appoint officers alone, but this statement of
fact does not reveal Lee’s point of view, nor does it match any of the answer choices for Q36.
Eliminate (37A). The lines for (37B) state that when a man has been authorized to exercise power
alone, he has never done so. This information does not match any of the answer choices for
Q36, so eliminate (37B). The lines for (37C) refer to the idea that distributing appointments
into several hands is essential to promoting beneficial appointments and preserving the balance in
government. This reveals Lee’s view and matches answer (36C). Draw a line connecting (36C)
and (37C). The lines for (37D) ask a rhetorical question about whether we have confidence in
our legislature. This does not match any of the answers for Q36, so eliminate (37D). Without
any support in the answers from Q37, (36A), (36B), and (36D) can be eliminated. The correct
answers are (36C) and (37C).

1 2   |   Practice Test 6: Answers and Explanations


SAT Premium Prep

37. C See explanation for Q36.

38. A The question asks how Hamilton would most likely have reacted to Lee’s remarks. First read Lee’s
remarks in lines 46–50: And it often happens when advice is wanted, the worst men, the most
interested creatures obtrude themselves, the worst advice is at hand, and misdirects the mind of
him who would be informed and advised. Q38 is the first question in a paired set, so it can be
done in tandem with Q39. Look at the answer choices for Q39. The lines for (39A) refer to the
idea that the president has the power to fill up all vacancies. This information does not provide
Hamilton’s view about the quality of advice given by a group to an individual, nor does it
match any of the answer choices for Q38. Eliminate (39A). The lines for (39B) refer to the idea
that a single man will have fewer personal attachments...than a body of men and will therefore
be less liable to be misled. This matches answer (38A). Draw a line connecting (38A) to (39B).
The line for (39C) state, This will be the usual condition of the bargain. This line doesn’t provide
any details about Hamilton’s view and doesn’t match any of the answers for Q38, so eliminate
(39C). The lines for (39D) state that the advancement of public service will rarely be the primary
object either of party victories or of party negotiations. At first glance, these lines may seem to
support (38A) since they are critical of a group’s decision-making. However, these lines don’t
compare the group’s decisions with an individual’s process, making (39B) a better match for
(38A). Eliminate (39D). Without any support in the answers from Q39, (38B), (38C), and
(38D) can be eliminated. The correct answers are (38A) and (39B).

39. B See explanation for Q38.

40. D This question asks about the relationship between the two passages. Because this is a question
about both passages, it should be done after the questions that ask about each passage indi-
vidually. The first paragraph of Passage 1 outlines the authority the president has to nominate
individuals for public office. Passage 1 goes on to argue that giving this authority to a single
individual has advantages over giving this authority to a group. The first paragraph of Passage
2 states that while the president may appoint officers alone, he never truly does. The second
paragraph of Passage 2 outlines the checks and balances to presidential power that the legis-
lature can exercise. The final paragraph of Passage 2 outlines the benefits a well formed disin-
terested legislature may provide. Therefore, the passages present alternative viewpoints: Passage
1 argues for the individual power of the president to appoint individuals, and Passage 2 advo-
cates for legislative assistance with such decisions. Eliminate answer choices that don’t match
this prediction. Passage 2 does not clarify information provided in Passage 1, so eliminate (A).
Passages 1 and 2 were both written in 1788, so one could not be characterized as contempo-
rary and the other as historical, so eliminate (B). Passage 1 presents an opinion, not a problem,
so eliminate (C). Passage 2 argues that the legislature should be consulted on presidential
appointments, while Passage 1 argues that decisions made by the president alone may be better
than those made by the legislature. Keep (D) because it matches the prediction. The correct
answer is (D).

Practice Test 6: Answers and Explanations  |   1 3


SAT Premium Prep

41. C The question asks about the main purpose of both passages. Because this is a question about
both passages, it should be done after the questions that ask about each passage individually.
Both passages discuss the power of the president to make individual appointments. Passage 1
argues that it is preferable for the president alone to make these appointments, while Passage 2
argues that the legislature should be consulted. Eliminate answer choices that don’t match this
prediction. The passages discuss how to choose people to fill offices; neither passage suggests
that creating political offices is a problem, so eliminate (A). Neither passage considers the best
way to establish a competent legislature, so eliminate (B). Both passages advocate an approach
for making political appointments, so keep (C). While Passage 2 touches on the government’s
system of checks and balances, Passage 1 does not. Eliminate (D). The correct answer is (C).

42. D The question asks for the main purpose of the passage. Since this is a general question, it should
be answered after the specific questions. The passage is about marsupial lions and evidence
of their presence in the Tight Entrance Cave. Eliminate answer choices that don’t match this
prediction. Eliminate (A) because the passage is not about marsupials in general. Eliminate
(B) because the passage is not about various Australian marsupials. Eliminate (C) because
the passage is not mainly about how researchers examine claw-mark fossil evidence. Keep (D)
because it matches the prediction. The correct answer is (D).

43. C The question asks how the focus shifts over the course of the passage. Since this is a general ques-
tion, it should be answered after the specific questions. It is the first question in a paired set,
so it can be done in tandem with Q44. Look at the answer choices for Q44 first. The lines for
(44A) describe how the species has been speculatively portrayed. There is no shift in these lines,
so eliminate (44A). The lines for (44B) define trace fossils, but there is no shift in these lines,
so eliminate (44B). The lines for (44C) say, The most parsimonious interpretation of the TEC
evidence is that marsupial lions were primarily responsible for the claw-mark assemblage. These
lines support (43C) since the lines contain the shift from physical evidence (the claw-mark
assemblage) to probable implications (the marsupial lions were primarily responsible). Draw a line
connecting (44C) to (43C). The lines for (44D) discuss the prey of the marsupial lions, the
rhinoceros-sized Diprotodon optatum. There is no shift in the lines, so eliminate (44D). Without
any support in the answers from Q44, (43A), (43B), and (43D) can be eliminated. The correct
answers are (43C) and (44C).

44. C See explanation for Q43.

45. A The question asks why the author uses the phrase “ its evocative depiction.” Use the given line
reference to find the window. Lines 5–7 say, The ubiquity of [T. carnifex] and its evocative depic-
tion in Aboriginal rock art suggest an important role in Australian ecosystems. Eliminate answer
choices that don’t match this prediction. Keep (A) because it matches the prediction. Choice
(B) is a Right Words, Wrong Meaning trap answer: the studies mentioned in the paragraph are
skeletal studies, not studies on the art. Eliminate (B). Eliminate (C) because there is no mention
of the depiction being a source of inspiration. Eliminate (D) because there is no indication that
the author admires the Aboriginal illustrations of Thylacoleo carnifex. The correct answer is (A).

1 4   |   Practice Test 6: Answers and Explanations


SAT Premium Prep

46. D The question asks what the phrase “Devils clearly used the cave” implies. Use the given line refer-
ence to find the window. Lines 36–39 say, The most parsimonious interpretation of the TEC
evidence is that marsupial lions were primarily responsible for the claw-mark assemblage and that
Devils clearly used the cave at times. Therefore, marsupial lions were not the only species to use
the cave. Eliminate answer choices that don’t match this prediction. Eliminate (A) because
the passage only mentions devils and marsupial lions, not several species of juvenile marsupials.
Choice (B) is a Could Be True trap answer: the passage says that devils used the cave, but not
that they were seen. Eliminate (B). Choice (C) is a Right Words, Wrong Meaning trap answer:
lines 40–43 say, it is improbable that the two species cohabited the chamber given the intense inter-
specific antagonism displayed by carnivores, especially in cave settings, not that the marsupial lions
eventually forced devils out. Eliminate (C). Keep (D) because it matches the prediction. The
correct answer is (D).

47. B The question asks for the most likely primary purpose once served by the Tight Entrance Cave.
This is the first question in a paired set, so it can be done in tandem with Q48. Look at the
answer choices for Q48 first. The lines for (48A) say that [T. carnifex’s] evocative depiction in
Aboriginal rock art suggest an important role in Australian ecosystems, but interpretations have
remained controversial. These lines do not mention the Tight Entrance Cave, so eliminate (48A).
The lines for (48B) say, Thousands of claw marks…are patchily distributed through the TEC main
chamber on a range of surfaces. Although these lines describe the Tight Entrance Cave, there is
no indication about the purpose of the cave. Eliminate (48B). The lines for (48C) say that the
distribution of claw marks indicates a significant bias toward juveniles, which suggests that the
cave was used as a den. Since the juveniles in discussion are marsupial lions, these lines support
(47B). Draw a line connecting (48C) and (47B). The lines for (48D) suggest that marsupial
lions were pack hunters. These lines do not mention the Tight Entrance Cave, so eliminate
(48D). Without any support in the answers from Q48, (47A), (47C), and (47D) can be elimi-
nated. The correct answers are (47B) and (48C).

48. C See explanation for Q47.

49. D The question asks which lines support the claim that the cave was not primarily inhabited by
adult marsupial lions. Look at the line references given in the answer choices and eliminate the
statements that don’t support this claim. The lines for (A) mention the claw-mark assemblage
in the main chamber of Tight Entrance Cave (TEC), but the lines do not discuss what inhabited
the caves, so eliminate (A). The lines for (B) say, The largest TEC ichnofossil scratch marks can
only have been made by T. carnifex. These lines indicate that marsupial lions (T. carnifex) did
inhabit the cave, so eliminate (B). The lines for (C) say that Devils clearly used the caves at times,
but this does not support the idea that the cave was not primarily inhabited by adult marsupial
lions as the devils were not the primary residents of the cave either. Eliminate (C). The lines for
(D) say that as individuals grew, they spent less time in the cave, presumably venturing out increas-
ingly to learn from adults and contribute to hunting. This implies that the majority of individuals
in the cave were juvenile marsupial lions, not adults, which supports the claim in the question.
The correct answer is (D).

Practice Test 6: Answers and Explanations  |   1 5


SAT Premium Prep

50. B The question refers to the figure and asks which inter-digital spacing measurement occurred with
the greatest frequency. Look at the figure to find the inter-digital spacing measurement that
occurred most often. The y-axis shows the frequency. The highest bar in the graph is at an inter-
digital spacing of 7 mm. Eliminate (A), (C), and (D) and keep (B). The correct answer is (B).

51. D The question asks for the inter-digital spacing of the claw marks that suggest the presence of
marsupial lions. Look at the figure to find information about marsupial lions. Based on the
passage, the scientific name for marsupial lions is Thylacoleo carnifex. The figure shows that the
inter-digital spacing of T. carnifex starts just above 20 mm and extends beyond 30 mm. Only
(D) reflects that range. Eliminate (A), (B), and (C) and keep (D). The correct answer is (D).

52. B The question asks which statement is best supported by information in both the passage and the
figure. First work though each answer choice using the figure. Eliminate (A) because the
figure does not provide information on the behavior of adult marsupial lions. Eliminate (C)
because the inter-digital spacing with the greatest frequency is 7 mm, and the marsupial lions
(T. carnifex) have inter-digital spacing of greater than 20 mm. Eliminate (D) because the figure
is about the frequency of the claw marks, not their shape. Choice (B) is supported by both the
passage and the figure: according to the figure, all species in the Tight Entrance Cave other than
Thylacoleo carnifex had inter-digital spacing of less than 20 mm, and the passage says that [d]
evils clearly used the cave at times. The correct answer is (B).

Section 2: Writing and Language


1. C Note the question! The question is asking which answer would set up the information that
follows, so it is testing consistency. Look for a word with a definition that is consistent with
the other ideas in the sentence. The sentence goes on to say that Salman Rushdie was born in
India and writes about Indian issues, and Doris Lessing was born in Iran and writes with an eye
to the international scene. In a paragraph about British authors, these details present interesting
complications to the term “British.” Choice (A) describes a simplicity present in the question,
which is irrelevant, so (A) can be eliminated. Choices (B) and (D) do not refer to any of the
information in the paragraph, so eliminate them. Choice (C) addresses the way that these
authors challenge obvious answers, so it effectively previews what is to come. The correct answer
is (C).

2. B Pronouns and nouns are changing in the answer choices, so this question is testing preci-
sion. A pronoun can only be used if it is clear what the pronoun refers to. Choices (A), (C),
and (D) present pronouns that could refer either to multiple things or nothing at all, so those
choices can be eliminated. Choice (B) refers to the specific noun, British issues, mentioned in
the previous sentence. The correct answer is (B).

1 6   |   Practice Test 6: Answers and Explanations


SAT Premium Prep

3. D Punctuation is changing in the answer choices, so this question is testing STOP, HALF-STOP,
and GO punctuation. Use the Vertical Line Test, and identify the ideas as complete or incom-
plete. The first part of the sentence, His first two novels, as one might expect, is an incomplete
idea. The second part, deal with Japanese characters and themes, is also an incomplete idea. To
connect two incomplete ideas, GO punctuation is needed. Eliminate (A) and (B) because a
semicolon is STOP punctuation and a dash is HALF-STOP punctuation. Keep (C) and (D)
because no punctuation and a comma are both GO punctuation. A comma comes after the
word novels, setting off the phrase as one might expect as unnecessary information. In order to
remain consistent with the non-underlined punctuation, there should be a comma after the
word expect as well. The correct answer is (D).

4. B Vocabulary is changing in the answer choices, so this question is testing precision of word
choice. Look for a word with a definition that is consistent with the other ideas in the sentence.
The sentence says that he wrote these novels about an “ imaginary Japan” that he’ d dreamt up, so
the correct word should mean “idea.” Mapping means “laying out,” so eliminate (A). Concep-
tion means “idea,” so keep (B). Transportation means “movement,” so eliminate (C). Inception
means “beginning,” so eliminate (D). The correct answer is (B).

5. D Transitions are changing in the answer choices, so this question is testing consistency of
ideas. A transition must be consistent with the relationship between the ideas it connects. The
sentence before the transition mentions his modest success after he wrote about Japanese charac-
ters and themes, and the next sentence mentions that he became a literary phenomenon after he
shifted to explicitly British themes. These are contrasting ideas, so eliminate (A) and (B), which
both contain same-direction transitions. But indicates a contrast but is not the right kind of
transition to appear at the beginning of a sentence and be followed by a comma, so elimi-
nate (C). Choice (D) appropriately uses the opposite-direction transition However. The correct
answer is (D).

6. D Note the question! The question asks whether the given sentence should be added, so it’s testing
consistency. If the content of the new sentence is consistent with the ideas surrounding it, then
it should be added. The paragraph discusses Ishiguro’s novel. The new sentence discusses the
film adaptation, so it is not consistent with the ideas in the text; the sentence should not be
added. Eliminate (A) and (B). Eliminate (C) because what is and is not British is not relevant to
the paragraph. Keep (D) because it accurately states that the new sentence does not contribute
to the paragraph. The correct answer is (D).

7. C Vocabulary is changing in the answer choices, so this question is testing precision of word
choice. The sentence already states the subject perspective, so the word it should not be used,
as it creates an incomplete sentence. Eliminate (A), (B), and (D). Keep (C) because it uses the
word that instead of it, creating a complete sentence. The correct answer is (C).

Practice Test 6: Answers and Explanations  |   1 7


SAT Premium Prep

8. C Note the question! The question asks which choice most effectively connects the sentence with the
previous paragraph, so it is testing consistency. Eliminate answers that are inconsistent with the
purpose stated in the question. The previous paragraph states that Ishiguro found his voice and
his subject matter when he shifted to explicitly British themes. This paragraph states that his most
recent novel…addresses the issue of British self-imagining even more directly. Look for an answer
choice that is consistent with the fact that he kept writing about the same ideas. Eliminate (A)
and (B) because they do not mention Ishiguro. Choice (C) is consistent with the paragraph,
mentions the author, and contains the phrase has since continued, which shows that he kept
writing the same thing; keep (C). Eliminate (D) because it discusses where the author was
raised and not what he wrote. The correct answer is (C).

9. D Note the question! The question asks which choice most effectively combines the underlined
sentences, so it’s testing consistency and concision. Eliminate answers that are inconsistent with
the purpose stated in the question. Eliminate (A) because it unnecessarily repeats the word
canvas. Eliminate (B) because the phrase it being is awkward and overly wordy. Eliminate (C)
because is a canvas and there on this canvas are redundant. Choice (D) combines the sentences
in a concise way without introducing any new errors. The correct answer is (D).

10. D Verbs are changing in the answer choices, so this question is testing consistency of verbs. A
verb must be consistent with its subject and with the other verbs in the sentence. The other
verb in the sentence is is, which is in the present tense. To be consistent, the underlined verb
must also be in the present tense. Eliminate (A) and (B), because was and had been are not in
present tense. Choice (C) is in the present tense but is not parallel to is; eliminate (C). Keep
(D) because is is in the present tense. The correct answer is (D).

11. B The length of the phrase after thinking is changing in the answer choices, so this question is
testing precision and concision. There is also the option to DELETE; consider this choice care-
fully as it is often the correct answer. First determine whether the phrase is necessary. A preposi-
tion is necessary, since the correct phrase is “thinking of” something, so eliminate (D). The non-
underlined portion of the sentence includes the words writer and he, so there is no need to repeat
these ideas. Eliminate (A) and (C). Choice (B) is consistent and concise. The correct answer is (B).

12. D The length of the phrase after from is changing in the answer choices, so this question is
testing precision and concision. There is also the option to DELETE; consider this choice
carefully as it is often the correct answer. First determine whether the phrase is necessary. The
non-underlined portion of the sentence draws a contrast between two items using the phrase
different from. The first item is human language, so the second item should be of the same type.
If the underlined portion is deleted, then the second item is the other calls, which is consistent
with human language. Therefore, (D) is the best option. Choice (A) contains those who speak,
which refers to speakers and is not consistent with human language. Eliminate (A). Choice (B)
contains speaking, which is a verb and is not consistent with the noun phrase human language.
Eliminate (B). Choice (C) contains animals who speak, which refers to speakers and is not
consistent with human language. Eliminate (C). The correct answer is (D).

1 8   |   Practice Test 6: Answers and Explanations


SAT Premium Prep

13. B Punctuation is changing in the answer choices, so this question is testing STOP, HALF-STOP
and GO punctuation. Use the Vertical Line Test and identify the ideas as complete or incom-
plete. Draw the vertical line between the words indicate and food. The first part of the sentence,
Alarm calls can cover any number of warnings or communications, which can indicate, is an
incomplete idea. The second part, food sources, mating, or predators, is also an incomplete idea.
To connect two incomplete ideas, GO punctuation is needed. Eliminate (D) because the colon
is HALF-STOP punctuation. There is no reason to use a comma after indicate, so eliminate
(A) and (C). The correct answer is (B).

14. C Verbs are changing in the answer choices, so this question is testing consistency of verbs. A
verb must be consistent with its subject and with the other verbs in the sentence. The subject
of the verb is the D call, which is singular. To be consistent, the underlined verb must also
be singular. Eliminate (B) and (D) because they are not singular. The verb in the previous
sentence is is used, which is in the present tense. To be consistent, the underlined verb must
also be in the present tense. Eliminate (A) because it is not in the present tense. The correct
answer is (C).

15. A Transitions are changing in the answer choices, so this question is testing consistency of
ideas. A transition must be consistent with the relationship between the ideas it connects. The
sentence after the transition says as interesting as this is, and ends with the bigger discovery was
yet to come. These are contrasting ideas, so eliminate (B), (C), and (D) which all contain same-
direction transitions. Choice (A) appropriately uses the opposite-direction transition Neverthe-
less. The correct answer is (A).

16. D Vocabulary is changing in the answer choices, so this question is testing precision of word
choice. Look for a word with a definition that is consistent with the other ideas in the sentence.
The sentence says Without even realizing it, which is an introductory phrase. Therefore, the
correct answer must include a subject and a verb to make the sentence complete; eliminate
(A) and (B), as they each contain a verb but no subject. Choice (C) contains a subject but no
verb. Eliminate (C). Choice (D) contains the subject humans and the verb rely, so it makes the
sentence complete. The correct answer is (D).

17. D Note the question! The question asks whether a sentence should be added, so it’s testing consis-
tency. If the content of the new sentence is consistent with the ideas surrounding it, then it
should be added. The paragraph discusses the great tit and its calls and makes no mention of
any other birds. The new sentence discusses the chickadee and its genus, so it is not consistent
with the ideas in the text; the sentence should not be added. Eliminate (A) and (B). Eliminate
(C) because the passage has not discussed chickadees. Keep (D) because it accurately states that
the new sentence is irrelevant. The correct answer is (D).

Practice Test 6: Answers and Explanations  |   1 9


SAT Premium Prep

18. C Transitions are changing in the answer choices, so this question is testing consistency of
ideas. A transition must be consistent with the relationship between the ideas it connects. The
sentence before the transition states that the birds would reliably respond to ABC-D calls, and
the sentence with the transition states that The birds would not respond to…an atypical combi-
nation of the call, D-ABC. These are contrasting ideas, so eliminate (A), (B), and (D), which
do not contain disagreeing transitions. Choice (C) appropriately uses the opposite-direction
transition however. The correct answer is (C).

19. A Vocabulary is changing in the answer choices, so this question is testing precision of word
choice. Look for a word with a definition that is consistent with the other ideas in the sentence.
The previous sentences say that the birds respond to ABC-D calls but would not respond to
D-ABC calls, so the answer should mean that order “is important.” Matters means “is impor-
tant,” so keep (A). Maintains and preserves both mean to “keep” or “sustain,” so eliminate (B)
and (D). Aligns means to “be the same as,” so eliminate (C). The correct answer is (A).

20. A Note the question! The question asks which choice would reinforce the paragraph’s claim about
the importance of syntax in bird calls, so it’s testing consistency. Eliminate answers that are
inconsistent with the purpose stated in the question. The paragraph states that the birds
respond to ABC-D calls but would not respond to D-ABC calls. Look for an answer choice that
is consistent with this idea. Choice (A) suggests that birds, like humans, don’t respond to out-
of-order or garbled communication. This is consistent with the idea that word order, or syntax,
is important, so keep (A). Eliminate (B) because, while it does mention word order, it doesn’t
mention its importance. Eliminate (C) and (D) because they do not mention that the order of
words is important. The correct answer is (A).

21. B Vocabulary is changing in the answer choices, so this question is testing precision of word
choice. All four choices mean the same thing in this context, but (B) is the most concise.
Eliminate (A), (C), and (D) because they are overly wordy. The correct answer is (B).

22. C Apostrophes are changing in the answer choices, so this question is testing apostrophe usage. When
used with a pronoun, an apostrophe indicates a contraction. In this sentence, the intended
meaning is figure out whether “there are” other animals. Choice (A) means there is other animals.
Eliminate (A) because the singular verb is is not consistent with the plural noun animals.
Choice (B) means they are is other animals, so eliminate it. Choice (C) correctly means there
are other animals, so keep it. Choice (D) means they are are other animals, so eliminate it. The
correct answer is (C).

23. A Note the question! The question asks which choice will convey an attitude of genuine interest
and avoid the appearance of sarcasm, so it’s testing consistency. Eliminate answers that are
inconsistent with the purpose stated in the question. Look for an answer choice that is consis-
tent with a formal tone and indicates a genuine interest. Choice (A) means “important” and
conveys a formal tone, so keep it. Eliminate (B) because it conveys an informal tone that isn’t
genuine. Eliminate (C) because it means “really big” and conveys an informal tone that isn’t

2 0   |   Practice Test 6: Answers and Explanations


SAT Premium Prep

genuine. Eliminate (D) because it means “generous” or “noble,” which is not consistent with
the meaning of the sentence. The correct answer is (A).

24. B Conjunctions are changing in the answer choices, so this question is testing STOP, HALF-
STOP, and GO punctuation. Use the Vertical Line Test, and identify the ideas as complete
or incomplete. Draw lines around the FANBOYS word and, since it is connecting the two
parts of the sentence. The first part of the sentence, Countless pollsters check in with likely voters
every day in the time before an election, is a complete idea. In (A), also predict which way the elec-
torate will tend on decision day, is an incomplete idea, so GO punctuation is needed. However,
the comma plus and is STOP punctuation, so eliminate (A). In (B), predicting which way the
electorate will tend on decision day, is an incomplete idea, so GO punctuation is needed. The
comma without the FANBOYS word is a type of GO punctuation, so keep (B). In (C), they
also predict which way the electorate will tend on decision day, is a complete idea, so STOP punc-
tuation is needed. However, the comma alone is GO punctuation, so eliminate (C). In (D),
predict which way the electorate will tend on decision day, is an incomplete idea, so GO punc-
tuation is needed. However, the comma plus but is STOP punctuation, so eliminate (D). The
correct answer is (B).

25. D Prepositions are changing in the answer choices, so this question is testing idioms. Look at the
word before the preposition to determine the correct idiom. Use process of elimination, and
guess if there is more than one answer left. The correct idiom is “planned to.” Eliminate (A)
and (C) because they do not use the correct idiom. Between the remaining answers, to chose in
(B) is an incorrect phrasing, so eliminate (B). The correct answer is (D).

26. C Note the question! The question asks which choice continues the idea in this sentence, so it’s
testing consistency. Eliminate answers that are inconsistent with the purpose stated in the
question. The first paragraph states that citizens have a sense of the likely outcome of that elec-
tion and that this knowledge comes from the opinion poll. This sentence says that the results
predicted the outcome correctly. Look for an answer choice that is consistent with these ideas.
Eliminate (A) because the polls will provide the people with knowledge but not tell the people
what to think. Eliminate (B) because the poll was correct and did not mislead anyone. To keep
the public informed is consistent with giving the public knowledge, so keep (C). Eliminate (D)
because to sell newspapers is new information and is not consistent with the information in the
surrounding paragraphs. The correct answer is (C).

27. B Vocabulary is changing in the answer choices, so this question is testing precision of word
choice. Choices (C) and (D) both include 1948 twice, so both answers can be eliminated
because they are repetitive. The first part of the sentence introduces a famous instance, and
the second part of the sentence describes the instance, so the connecting phrase should indi-
cate a same-direction transition. Eliminate (A) because yet is an opposite-direction transition.
Choice (B) appropriately uses the same-direction transition when. The correct answer is (B).

Practice Test 6: Answers and Explanations  |   2 1


SAT Premium Prep

28. B The punctuation is changing in the answer choices, so this question is testing STOP, HALF-
STOP, and GO punctuation. Use the Vertical Line Test, and identify the ideas as complete
or incomplete. Draw the vertical line between the words election and by. The first part of the
sentence, Once all the ballots were cast, it had become clear that in fact Truman had won the elec-
tion, is a complete idea, and the second part, By that time polling had earned a statistical sophisti-
cation that had been unrivalled in earlier eras, is also a complete idea. To connect two complete
ideas, STOP or HALF-STOP punctuation is needed. The period is STOP punctuation and
the dash is HALF-STOP punctuation, so keep (A), (B), and (C). Eliminate (D) because no
punctuation is GO. Next, the punctuation between time and polling is changing, so use the
Vertical Line Test again. The first part of the sentence, By that time, is an incomplete idea, and
the second part, polling had earned a statistical sophistication that had been unrivalled in earlier
eras, is a complete idea. To connect an incomplete idea to a complete idea, GO punctuation
is needed. Eliminate (A) because a colon is HALF-STOP punctuation. Keep (B) because a
comma is GO punctuation. Eliminate (C) because a semicolon is STOP punctuation. The
correct answer is (B).

29. A Note the question! The question asks which choice will correspond as closely as possible with the
information in the map, so it’s testing consistency. Eliminate answers that are inconsistent with
the map and its legend. The sentence states that Truman carried these states. Look for an answer
choice in which all of the states listed are dark gray. CA to NV to OH is the only list in which
all the states listed were won by Truman. The correct answer is (A).

30. D Apostrophes are changing in the answer choices, so this question is testing apostrophe usage.
When used with a pronoun, an apostrophe indicates a contraction. The intended meaning of
the first part of the phrase is it is only as good as…, so an apostrophe is needed. Eliminate (A)
and (C). Next, look at the words after the underlined portion. Choice (B) would say “it is”
sample size, but the intended meaning is the possessive its, meaning “belonging to the poll.”
Eliminate (B) because it is the wrong form of the possessive pronoun. The correct answer is
(D).

31. B Note the question! The question asks where the new sentence should be placed, so it’s testing
consistency of ideas. The sentence must be consistent with the ideas that come both before and
after it. Determine the subject matter of the sentence, and find the other sentence(s) that also
discuss that information. The new sentence mentions those 100 people, so because of the word
those, it should come after a previous reference to 100 people. This is mentioned in sentence
2, so eliminate (A). The new sentence also mentions another one, so it needs to come after a
mention of something that another one could refer back to. Sentence 3 states that you’ d get one
answer, so the new sentence would correctly follow this idea by referencing another answer that
you’ d get. Therefore, the new sentence should follow sentence 3. The correct answer is (B).

2 2   |   Practice Test 6: Answers and Explanations


SAT Premium Prep

32. C Verbs are changing in the choices, so this question is testing consistency of verbs. A verb must
be consistent with its subject and with the other verbs in the sentence. The subject of the verb
is voters, which is plural. To be consistent, the underlined verb must also be plural. Eliminate
(B) and (D) because their verbs are singular. The sentence says even today, so the verb must be
in present tense. Eliminate (A) because it is not in present tense. The correct answer is (C).

33. A Note the question! The question asks which choice concludes the sentence and paragraph, so it’s
testing consistency of ideas. Eliminate answers that are inconsistent with the purpose stated
in the question. The passage discusses how polls are used to predict public opinion. Look for
an answer choice that is consistent with polls predicting public opinions. Keep (A) because
it responds to the previous questions by asking for an opinion and a poll, so it is consistent.
Eliminate (B) because it describes the polls as harmful, which is not consistent. Eliminate
(C) because it says Who cares…?, which is inconsistent with the author’s tone. Eliminate (D)
because it says it’s time to get rid of them, which is also inconsistent with the author’s tone. The
correct answer is (A).

34. D Pronouns are changing in the answer choices, so this question is testing consistency of
pronouns. A pronoun must be consistent in number with the noun or pronoun it refers to.
The underlined pronoun refers to someone, which is singular. To be consistent, the underlined
pronoun must also be singular. Eliminate (A), (B), and (C) because they contain the plural
pronouns they, we, and their. The correct answer is (D).

35. A Verbs are changing in the answer choices, so this question is testing consistency of verbs. A
verb must be consistent with its subject and with the other verbs in the sentence. The other
verb in the sentence is is, which is in the present tense. To be consistent, the underlined verb
must also be in the present tense. Eliminate (B) and (C) because they are not in the present
tense. The subject of the verb is public eye, which is singular. To be consistent, the underlined
verb must also be singular. Eliminate (D) because it is plural. The correct answer is (A).

36. C Transitions are changing in the answer choices, so this question is testing consistency of ideas.
A transition must be consistent with the relationship between the ideas it connects. The first
part of the sentence says this may sound like a joke, and the second part of the sentence says
it’s not at all. These are contrasting ideas, so eliminate (B) and (D), which both contain same-
direction transitions. Although but does indicate a contrast, it makes the sentence incomplete,
so eliminate (A). Choice (C) appropriately uses the opposite-direction transition while and
makes the sentence complete. The correct answer is (C).

37. C Punctuation is changing in the answer choices, so this question is testing STOP, HALF-
STOP, and GO punctuation. Use the Vertical Line Test, and identify the ideas as complete or
incomplete. Draw the vertical line between the words restaurant and for. The first part of the
sentence, Imagine a new business opens in your city, a restaurant, is a complete idea. The second
part, for instance, food-critic reviews can be difficult to come by, and after all, how powerful can
a food critic’s review actually be in most places, is a complete idea. To connect two complete

Practice Test 6: Answers and Explanations  |   2 3


SAT Premium Prep

ideas, STOP or HALF-STOP punctuation is needed, so (A) may seem to work. However, the
meaning is not precise in this construction: it implies that the second part of the sentence is
an example of the first, which isn’t correct. Eliminate (A). Next, (C) has a period after instance,
so do the Vertical Line Test again, in that location. In this case, the first part of the sentence
is Imagine a new business opens in your city, a restaurant, for instance, which is complete. The
second part of the sentence is Food-critic reviews can be difficult to come by, and after all, how
powerful can a food critic’s review actually be in most places, which is also complete. Therefore,
STOP or HALF-STOP is needed, so eliminate (B) and (D). Choice (C) correctly punctuates
the ideas and creates a precise meaning, as for instance should go with the first idea—a restau-
rant is an example of a business that could open. The correct answer is (C).

38. A Punctuation is changing in the answer choices, so this question is testing STOP, HALF-STOP,
and GO punctuation. The answer choices have combinations of multiple punctuation marks,
so consider each one individually. Choice (A) uses two dashes to separate a list of positive feed-
back types. Without the phrase, the sentence reads They might generate positive feedback on that
restaurant’s Facebook page. This is a complete and precise sentence, so it implies that the infor-
mation set off in dashes is unnecessary, which implies that (A) works, so keep it. Choice (B)
uses two semicolons, but semicolons only link complete ideas. The sentence does not contain
three complete ideas, so eliminate (B). Choices (C) and (D) both use a colon, but that can’t
work because the part after the comma contains both an unnecessary list and the completion
of the main idea of the sentence. A colon is used when the part after it provides a related expla-
nation or definition and nothing else. Eliminate (C) and (D). The correct answer is (A).

39. B Note the question! The question asks where sentence 2 should be placed, so it’s testing consis-
tency of ideas. The sentence must be consistent with the ideas that come both before and after
it. Sentence 2 introduces the Fake Facebookers, so it must come before more information about
this group. Sentence 3 says that the owner will create a powerful social-media presence instead
of relying on the food critic. Sentence 4 says They might generate positive feedback and must be
referring to the Fake Facebookers, as there is no plural noun in the previous sentence that they
could refer back to. Therefore, sentence 2 should follow sentence 3. The correct answer is (B).

40. A Note the question! The question asks whether a sentence should be added, so it’s testing
consistency. If the content of the new sentence is consistent with the ideas surrounding it,
then it should be added. The paragraph states that One option is to post pictures of you and a
new partner having a great time and mentions this style of jealousy-inducing revenge. The new
sentence discusses a breakup and how to make him or her jealous, so it is consistent with the
ideas in the text; the sentence should be added, since it introduces the idea that the paragraph
elaborates on. Eliminate (C) and (D). The information in the new sentence is discussed further
in the paragraph, so keep (A). Eliminate (B) because the breakup is not discussed throughout the
passage. The correct answer is (A).

2 4   |   Practice Test 6: Answers and Explanations


SAT Premium Prep

41. C Note the question! The question asks which choice best supports the point developed in this
paragraph, so it’s testing consistency. Eliminate answers that are inconsistent with the purpose
stated in the question. The paragraph discusses using Fake Facebookers to show any social aspect
of your personality. Look for an answer choice that is consistent with the idea that the Fake
Facebookers help one show something. Eliminate (A) because to provide solace is not to show.
Eliminate (B) because to provide harmful effects is not to show. Keep (C) because to provide
evidence matches to show. Eliminate (D) because to provide a profitable enterprise is not to show.
The correct answer is (C).

42. B Vocabulary is changing in the answer choices, so this question is testing precision of word
choice. Look for a word with a definition that is consistent with the other ideas in the sentence.
The sentence says that the ethics are irrelevant, so the correct word should mean “to judge.”
Adore means “to love,” so eliminate (A). Assess means “to determine,” which can mean to judge,
so keep (B). Obsess means “to dwell on,” so eliminate (C). Philosophize means “to theorize,” so
eliminate (D). The correct answer is (B).

43. B The length of the phrase after because is changing in the answer choices, so this question is
testing precision and concision. Choice (A) misplaces the word culture. The avatars are not of
culture, so eliminate (A). Choice (B) correctly places culture to modify internet avatars, so keep
(B). Choice (C) says the strong culture…has so much power, which is redundant, so eliminate
(C). Choice (D) says the powerful culture…is full of strength, which is also redundant. Elimi-
nate (D). The correct answer is (B).

44. C Vocabulary is changing in the answer choices, so this question is testing precision of word
choice. The sentence starts with the verb phrase shuddering or not, so the subject of the verb
shuddering must immediately follow the comma. The previous sentence states We may shudder,
so the subject is we. Choices (A), (B), and (D) do not begin with we, so eliminate them. The
correct answer is (C).

Section 3: Math (No Calculator)


1. B The question asks for the value of a function at a specific x-value. First, find the value of
the constant c. The question states that g(15) = 2, so plug 15 in for x in the function and set
2 30
the result equal to 2. The equation becomes 2 = (15) + c . This simplifies to 2 = + c or 2
5 5
2
= 6 + c. Subtract 6 from both sides of the equations to get c = –4. Therefore, g ( x ) = x − 4 .
5
2
The question asks for g(–5). Plug –5 in for x to get g( −5) = ( −5) − 4 , which simplifies g(–5) =
5
–2 – 4 = –6. The correct answer is (B).

Practice Test 6: Answers and Explanations  |   2 5


SAT Premium Prep

2. B The question asks for the value of x in a system of equations. Since the question asks for a

specific value and the answers contain numbers in increasing order, plug in the answers. Begin

by labeling the answers as “x” and start with (B), 3. Plug x = 3 into the first equation to get

9(3 + 4) = y. This becomes 9(7) = y or 63 = y. Now check to see whether these values of x and y
y 63
satisfy the second equation. If = 21 , then = 21 , which is true, so stop here. The correct
x 3
answer is (B).

3. D The question asks which equation is true for some value of a. Taking the absolute value of an
expression will yield a positive result. Isolate the absolute value in each choice, and eliminate
any that result in a negative value for the absolute value. Subtract 1 from both sides of (A) to
get |a – 1| = –1. This cannot be true, so eliminate (A). Subtract 1 from both sides of (B) to get
|1 – a| = –1. Eliminate (B). Subtract 1 from both sides of (C) to get |a + 1| = –1. Eliminate (C).
Add 1 to both sides of (D) to get |a – 1| = 1. Since the absolute value is set equal to a positive
number, there is a value of a that could make this true. Choice (D) is true if a = 0 or a = 2. The
correct answer is (D).

4. A The question asks which equation must be true given an initial equation. There are variables

in the answers, so plug in. Start by setting the denominators equal by making y = 5. Then set

the numerators equal to get x – y = 2. Plug in y = 5 to get x – 5 = 2. Add 5 to both sides of the

equation to get x = 7. Therefore, x = 7 and y = 5 satisfy the equation. Plug these values into each
7 7
of the choices and eliminate any that are not true. Choice (A) becomes = . This is true, so
5 5
7 3
keep (A), but check the remaining answers just in case. Choice (B) becomes 5 = − 5 . Elimi-
7 − 2(5) 8 7 − 10 8 3 8
nate (B). Choice (C) becomes = − , which simplifies to = − or − = − .
5 5 5 5 5 5
7 +5 7 12 7
Eliminate (C). Choice (D) is = or = . Eliminate (D). The correct answer is (A).
5 5 5 5
5. C The question asks for the result when plugging an expression into an equation. Because there
are variables in the answer choices, plug in. Make x = 3. If x = 3, then –2x = –2(3) = –6. There-
fore, h(–2x) = h(–6) = –5(–6) + 3 = 30 + 3 = 33. This is the target value; circle it. Now plug
x = 3 into the answer choices to see which one matches the target value. Choice (A) becomes
–10(3) – 3 = –30 – 3 = –33. This does not match the target, so eliminate (A). Choice (B)
becomes 10(3) – 3 = 30 – 3 = 27. Eliminate (B). Choice (C) becomes 10(3) + 3 = 30 + 3 = 33.
Keep (C), but check (D) just in case. Choice (D) becomes 10(3)2 – 6(3) = 10(9) – 18 = 90 – 18
= 72. Eliminate (D). The correct answer is (C).

2 6   |   Practice Test 6: Answers and Explanations


SAT Premium Prep

6. D The question asks for an equivalent form of an expression. One approach to this question is to
use Bite-Sized Pieces. First determine the x 2 term. Multiply the two terms with x to get (9x)
(3x) = 27x 2. Multiply this by the coefficient in front to get 2(27x) = 54x 2. Eliminate (A) and (B)
because they do not have 54x 2. Now look at the remaining choices. Choice (D) includes 24x
while (C) has no x term. Determine whether the x term could cancel. Terms could only cancel
if they subtract out. Since all terms are positive, this cannot happen, so eliminate (C). Only
one choice remains. Because there are variables in the choices, another approach is to plug in
for x. Make x = 2. If x = 2, then 2(9x + 1)(3x + 1) = 2[9(2) + 1][3(2) + 1] = 2(19)(7) = 38(7) =
266. This is the target value; circle it. Now plug x = 2 into the answer choices to see which one
matches the target value. Choice (A) becomes 80(2) = 160. This does not match the target, so
eliminate (A). Choice (B) becomes 12(2)2 + 4 = 12(4) + 4 = 48 + 4 = 52. Eliminate (B). Choice
(C) becomes 54(2)2 + 2 = 54(4) + 2 = 216 + 2 = 218. Eliminate (C). Choice (D) becomes
54(2)2 + 24(2) + 2 = 216 + 48 + 2 = 266, so keep (D). The correct answer is (D).

7. C The question asks for the solution set of an equation. First, plug k = 3 into the equation to
get x − 3 = x − 9 . Since the answers contain numbers, plug in the answers. Two of the solu-
tion sets involve 7 and 12, so pick one of these values and plug in. Plug in x = 7. The equation
becomes 7 − 3 = 7 − 9 or 4 = −2 , which is not true. Eliminate (B) and (D) because they
both contain 7. Plug in one of the two remaining choices. Try (C), 12. The equation becomes
12 − 3 = 12 − 9 or 9 = 3 . Since this is true, x = 12 is a solution. The correct answer is (C).

8. B The question asks which statement best describe Kyung’s deposits. Kyung had a balance of

$5,000 after 5 months and $5,800 after 21 months. First, subtract to find the number of

months between 5 and 21 months to get 21 – 5 = 16 months. Next, find the total amount

deposited over these 16 months by subtracting the balance at 5 months from the balance at 21

months to get $5,800 – $5,000 = $800 deposited. Finally, divide $800 by 16 months to get
$800
the amount deposited each month, or = $50 . The correct answer is (B).
16

9. D The question asks for the equation of a line that is parallel to another line. When two lines are

parallel, they have equal slopes. To determine the slope of a line given the equation, put the

equation into slope-intercept form: y = mx + b, where m is the slope. The equation in the ques-

tion is already in y = mx + b form, so the slope of the line represented by y = –5x + 9 is –5. Put

the equations in the answer choices in y = mx + b form, and find a choice that has a slope equal

to –5. In (A), subtract x from both sides of the equation to get 5y = –x + 1. Divide both sides
1 1 1
by 5 to get y = − x + . The slope is − . This does not equal –5, so eliminate (A). In (B),
5 5 5

Practice Test 6: Answers and Explanations  |   2 7


SAT Premium Prep

subtract 3x from both sides of the equation to get –5y = –3x + 15. Divide both sides by –5 to
3 3
get y = x − 3 . The slope is . Eliminate (B). In (C), subtract 5x from both sides of the equa-
5 5
tion to get –y = –5x + 3. Divide both sides by –1 to get y = 5x – 3. The slope is 5. Eliminate (C).

In (D), subtract 15x from both sides of the equation to get 3y = –15x + 12. Divide both sides by

3 to get y = –5x + 4. The slope is –5. The correct answer is (D).

10. B The question asks for the number of solutions to a set of equations. Questions about the

number of solutions to a system of equations are asking for the number of points of intersec-

tion. Since the first equation is a quadratic, which represents a parabola, and the second equa-

tion is a line, there can be at most two points of intersection. Eliminate (A). Solving this system

by substitution would be difficult, so sketch the equations. Rearrange the second equation into 

y = mx + b form. Subtract 7 from both sides of the equation to get x – 7 = 3y, and flip the equa-
1 7
tion to get 3y = x – 7. Divide both sides by 3 to get y = x − . The line will have a y-intercept
3 3
7
of − , which is between –2 and –3, and a shallow slope, looking like this:
3

1 7
y= x 
3 3

2 8   |   Practice Test 6: Answers and Explanations


SAT Premium Prep

2
Now sketch the parabola. Set y = 0 to find the x-intercepts. If 0 = (3x – 2)(x + 4), then x =
3
or –4. Now, to find the y-intercept, plug in x = 0. The equation becomes y = [3(0) – 2](0 + 4) =
(–2)(4) = –8. Add these intercepts to the sketch. The vertex of the parabola will be between the
two x-intercepts, somewhere to the left of and below the y-intercept. Sketch the parabola by
connecting the dots.

1 7
y= x 
3 3

y = (3x – 2)(x + 4)

The line and the parabola intersect twice; therefore, there are two solutions. The correct answer
is (B).

11. B The question asks for an expression representing the amount paid by each of Gary and Ron
for the radio. There are variables in the answer choices, so plug in. Make d = 20. The price of
Gary’s radio becomes $20, and the price of Ron’s radio becomes $20 + $10 = $30. Therefore,
the radios together cost $20 + $30 = $50. The taxes and shipping fees are 25% of the cost of
25 1 50
the radios alone. This is × 50 = × 50 = = $12.50 . Add this to the total cost to
100 4 4
get $50 + $12.50 = $62.50. The question asks what each of them paid. They agree to equally
divide the total cost, so they each pay $62.50 ÷ 2 = $31.25. This is the target value; circle it.
Now plug d = 20 into the answer choices to see which one matches the target value. Choice (A)
becomes 0.25(20) + 2.5 = 5 + 2.5 = $7.50. This does not match the target value, so eliminate
(A). Choice (B) becomes 1.25(20) + 6.25 = 25 + 6.25 = $31.25. Keep (B), but check the rest
of the choices just in case. Choice (C) becomes 1.75(20) + 10 = 35 + 10 = $45. Eliminate (C).
Choice (D) becomes 2.5(20) + 12.5 = 50 + 12.5 = $62.50. Eliminate (D). The correct answer is (B).

Practice Test 6: Answers and Explanations  |   2 9


SAT Premium Prep

12. A The question asks for the value of y in an equation. Although there are numbers in the choices,
the numbers are complicated and this is the no-calculator section, so do not plug in the answers.
Instead solve algebraically. First, get rid of the denominator by multiplying each side of the equa-
tion by (y – 3) to get y + 3 = 6(y – 3). Distribute the 6 on the right side to get y + 3 = 6y – 18. Add
18 to both sides of the equation to get y + 21 = 6y. Subtract y from both sides to get 21 = 5y. Divide
21
both sides by 5 to get = y . The correct answer is (A).
5

13. A The question asks for the values for x in an equation. The answer choices are in the form of the

−b ± b 2 − 4 ac
quadratic equation, which is x = for quadratics in the form ax 2 + bx + c = 0.
2a
Put the equation in the question into ax 2 + bx + c = 0 form by subtracting 3n from both sides of the

m
equation to get x 2 + x − 3n = 0 . To avoid plugging a fraction into the quadratic formula, multiply
3
everything in the equation by 3 to get 3x 2 + mx – 9n = 0. In this form, a = 3, b = m, and c = –9n.

−m ± m 2 − 4(3)( −9n ) −m ± m 2 + 108n


Plug these into the quadratic formula to get x = or x =
2(3) 6
2
m m + 108n
. The choices break this into two fractions, so do the same to get x = − ± . The
6 6
correct answer is (A).

14. C The question asks for the value of c, which, according to the figure, is the x-coordinate of the point

of intersection on the positive x-axis. One way to find the point of intersection is to set the two

equations equal and solve for x. However, in this case, that is unnecessary since the y-coordinate

is known to be 0. Set either one of the two equations equal to 0. If p(x) = 0, then 27x 2 – 3 =
3
0. Add 3 to both sides of the equation to get 27x 2 = 3. Divide both sides by 27 to get x 2 =
27
1
2
. Reduce the fraction by 3 to get x = . Take the square root of both sides of the equation to
9
1 1 1 1
get x = ± =± = ± . The question asks for the value of c, which is positive, so c = 3 . The
9 9 3
correct answer is (C).

3 0   |   Practice Test 6: Answers and Explanations


SAT Premium Prep

15. C The question asks for a coefficient of an imaginary number given as a fraction with imaginary

parts. To simplify such a fraction, multiply the numerator and denominator by the conjugate of

the base. The conjugate is the same expression but with the sign of the imaginary part changed.

The conjugate of 4 – 3i is 4 + 3i. Multiply the numerator and denominator by this expression

10 + 5i 4 + 3i
to get × . Multiply the fractions by multiplying the numerators and denomina-
4 − 3i 4 + 3i
10( 4 ) + 10(3i ) + 5i( 4 ) + 5i(3i )
tors separately. In both cases, use FOIL to get . Simplify to
4( 4 ) + 4(3i ) + ( −3i )( 4 ) + ( −3i )(3i )
40 + 30i + 20i + 15i 2 40 + 50i + 15i 2
get . Combine like terms to get . Since i2 = –1, the expres-
16 + 12i − 12i − 9i 2 16 − 9i 2
40 + 50i + 15( −1) 40 + 50i − 15 25 + 50i
sion equals , which becomes or . Factor out 25 in the
16 − 9( −1) 16 + 9 25

numerator to get 25(1 + 2i ) , then cancel the 25 in the numerator and the 25 in the denominator
25
to get 1 + 2i. The question says that this expression is written in the form a + bi. Since 1 + 2i =

a + bi, a = 1 and b = 2. The question asks for the value of b, which is 2. The correct answer is (C).
8 4
16. , , or 0.8
10 5

The question asks for the value of the sine of angle a. There are trigonometric expressions involved,

so write out SOHCAHTOA to remember the trig functions. The value 0.8 is equivalent to the frac-
8 adjacent adj 8
tion . The CAH part defines the cosine as , so cosb = = . Plug in 8 for the
10 hypotenuse hyp 10
adjacent side (the base of the triangle) and 10 for the hypotenuse. This is a 6-8-10 right triangle,
opposite
making the opposite side 6. The question asks for sin a. The SOH part defines the sine as hypotenuse .
opp 8
The side opposite angle a is 8, and the hypotenuse is 10, so sin a = = . The correct answer
hyp 10
8 4
can be expressed as , , or 0.8.
10 5

17. 30 The question asks for the value of c, which is the constant decrease in restoring force for each

additional 20 centimeters of stretching. When the rubber band is stretched from 30 centime-

ters to 90 centimeters, the restoring force decreases from –35 Newtons to –125 Newtons. Find

the distance stretched and the difference between the restoring force at each stretched distance.

There is a change in stretched distance of 90 – 30 = 60 centimeters, and there is a decrease in

Practice Test 6: Answers and Explanations  |   3 1


SAT Premium Prep

the restoring force of –35 – (–125) = 90 Newtons. The question asks for the decrease in restoring

force when the rubber band is stretched an additional 20 centimeters, so set up the proportion
c Newtons 90 Newtons
= . Cross-multiply to get 60c = 1,800. Divide both sides of the
20 centimeters 60 centimeters
equation by 60 to get c = 30. The correct answer is 30.

18. 12 The question asks for the maximum height of a book on the bottom shelf. The question states that
the shelves are all parallel. All the smaller triangles within the big triangle contain the same angle
at the top. Because the shelves are all parallel, these smaller triangles and the big triangle also
contain the same angle on the right side. Therefore, the shelves form four similar triangles. Use the
fact that similar triangles have proportional sides. As a result, the heights of the four shelves are in
the same proportion as the slant height of the four shelves. On the right side of the figure, the slant
heights of the four shelves are listed as x, 2x, 3x, and 2x. Therefore, the proportion of the heights is
also 1:2:3:2. Since the height of the entire bookcase is 48 inches, set up the equation y + 2y + 3y +
2y = 48. Combine like terms to get 8y = 48. Divide both sides of the equation by 8 to get y = 6. The
question asks for the maximum height of a book on the bottom shelf, which is 2y = 2(6) = 12. The
correct answer is 12.

19. 2 The question asks for the real value of x in an equation. To solve this, factor by grouping. Factor
the first two terms and the last two terms separately. The first two terms, x3 – 2x 2, factor to
x 2(x – 2). The last two terms, 3x – 6, factor to 3(x – 2). The equation can be rewritten as x 2(x – 2) +
3(x – 2) = 0. Since both of the new terms have a factor of (x – 2), factor this term out to get (x – 2)
(x 2 + 3) = 0. To find the solutions, set both of the factors equal to 0. Set the second factor equal
to 0 to get x 2 + 3 = 0. Subtract 3 from both sides to get x 2 = –3. Since the square of a real number
cannot be negative, this has no real solutions. Set the other factor equal to 0 to get x – 2 = 0. Add
2 to both sides of the equation to get x = 2. The correct answer is 2.

20. 5 The question asks for the value of b, the y-coordinate of a solution to a system of equations. To
solve a system of equations, stack and then add or subtract. First, though, it is necessary to multiply
both sides of both equations by a constant in order to cancel one of the variables. The question
asks for the value of b, so cancel out the a’s. Make the coefficients of a the same number with
opposite signs in the equations by multiplying the first equation by 6 and the second by –5. The
two equations become 30a – 12b = –60 and –30a – 20b = –100, respectively. Now stack and add
the equations.

30a – 12b = –60


–30a – 20b = –100
–32b = –160

Divide both sides of the equation by –32 to get b = 5. The correct answer is 5.

3 2   |   Practice Test 6: Answers and Explanations


SAT Premium Prep

Section 4: Math (Calculator)


1. A The question asks for an expression that models a specific situation. Translate English to math using
bite-sized pieces. The question states that Yoonhee’s longest run was 23 kilometers. The number 23
represents the initial value. Eliminate (C) and (D) since the initial value is not 23. The question also
states that Yoonhee’s training plan increases...her longest run by 2 kilometers per week. This indicates
that the value 2w should be added to the run, not subtracted. Eliminate (B). The correct answer is (A).

2. A The question asks for a measurement and gives conflicting units. The question states that the baker
separates a 6-pound batch of dough in half. Divide by 2 to get the weight of half the dough, or 6 ÷ 2
= 3 pounds. When dealing with scale maps or models, make a proportion, being sure to match up
1 pound 3 pounds
units. The proportion is = . Cross-multiply to get x = 48 ounces. Each half
16 ounces x ounces
batch of dough makes 12 donuts. Divide the number of ounces to find the weight per donuts, or
48 ÷ 12 = 4 ounces per raw donut. The correct answer is (A).

3. C The question asks for the number of tournaments Kitty participated in during a specific month.
Since the question asks for a specific value and the answers contain numbers in increasing order,
plug in the answers. Begin by labeling the answers as “# of tournaments” and start with (B), 3. The
question states that the cost of participating in tournaments is an additional $2.50 per tournament.
Multiply the number of tournaments by the cost per tournament to get 3 × $2.50 = $7.50. To get
the total cost, add the monthly subscription fee to the cost of participating in tournaments to get
$14.95 + $7.50 = $22.45. Since this does not match Kitty’s cost of participating, eliminate (B). Try
(C), 4. Multiply the number of tournaments by the cost per tournament to get 4 × $2.50 = $10.00.
To get the total cost, add the monthly subscription fee to the cost of participating in tournaments
to get $14.95 + $10.00 = $24.95. This matches the value given in the question, so stop here. The
correct answer is (C).

4. C The question asks for the number of necklaces Nathalie made last month. Since the ques-
tion asks for a specific amount and the answers contain numbers in increasing order, plug
in the answers. Begin by labeling the answers as “necklaces made by Nathalie” and start
with (B), 37. The question states that Aaron made 14 fewer necklaces than Nathalie. If
Nathalie made 37 necklaces, then Aaron made 37 – 14 = 23 necklaces. Add the number of
necklaces Nathalie made and the number of necklaces Aaron made to get 37 + 23 = 60 neck-
laces. This does not match the total number of necklaces made, so eliminate (B). Try (C), 44. If
Nathalie made 44 necklaces, then Aaron made 44 – 14 = 30 necklaces. Add the number of necklaces
Nathalie made and the number of necklaces Aaron made to get 44 + 30 = 74 necklaces. This
matches the value given in the question, so stop here. The correct answer is (C).

Practice Test 6: Answers and Explanations  |   3 3


SAT Premium Prep

5. B The question asks about population based on information about a study of a sample from that
population. Since the members were randomly selected, the number of members who prefer
attending the Montreal event found in the study should match that of the larger population. To
extrapolate the study results, multiply the given percent by the total number of people in the club
39.3
to get × 80 = 31.44 . The correct answer is (B).
100

6. C The question asks for the value of the force when given other information. Translate the infor-

mation in bite-sized pieces. One piece of information says that the distance traveled is equal to

the work expended divided by the force. Make the distance traveled D, work expended W, and
W
force F. The equation becomes D = . Next, plug in the given information. The work (W ) is
F
36
36 Newton-meters, and the distance (D) is 9 meters, so the equation becomes 9 = . Multiply
F
both sides of the equation by F to get 9F = 36. Divide both sides of the equation by 9 to get

F = 4. The correct answer is (C).

# of outcomes that fit requirements


7. D The question asks for a probability, which is defined as .
total # of outcomes
Read the table carefully to find the numbers to make each probability. In 2010, there were

1,298,300 total employees, so that is the total # of outcomes. Of those employees, 150,890

employees are in the Service category and 623,320 employees in the Office category. Add

both numbers together to get a total of 150,890 + 623,320 = 774,210 employees in both

categories, so that is the # of outcomes that fit requirements. Therefore, the probability of
774, 210
choosing an employee in either the Service category or the Office category is < 0.60 .
1, 298, 300
The correct answer is (D).

8. D The question asks for a proportion of the number of novels published by a certain publisher during
a specific range. To find the proportion, set up a ratio of the novels published by Doubleday divided
by the total number of books published. Asimov had 2 novels published by Doubleday in the years
2 1
1960-1979, and he had 40 novels published in total, so the ratio becomes = . The correct
40 20
answer is (D).

3 4   |   Practice Test 6: Answers and Explanations


SAT Premium Prep

9. C The question asks for a statement that must be true if a line has a positive slope in the xy–plane.
Use process of elimination to eliminate answer choices that are either false or could be true some of
the time, but not all the time. First, draw the xy-plane with the quadrants labeled.

II I

III IV

A line with a positive slope passes through Quadrants I and III since lines in the xy-plane go
to infinity in both directions. Eliminate (A) and (B) because they do not contain both Roman
numerals I and III. Next determine if Roman numeral II must be true. A line with a negative
y-intercept will not intersect Quadrant II. For example:

II I

III IV

Therefore, Roman numeral II is not necessarily true since a line with a positive slope will not
include points in Quadrant II when the line has a negative y-intercept. Eliminate (D) since it
contains Roman numeral II. The correct answer is (C).

Practice Test 6: Answers and Explanations  |   3 5


SAT Premium Prep

10. D The question asks for the equation of a function given the x-intercepts. If h(x) has x-intercepts at
−2, 2, and 4, then the function will have factors of (x – 2), (x + 2), and (x – 4). Eliminate answer
choices (A), (B), and (C) since the equations do not have all three factors. The correct answer is (D).

11. B The question asks for a value based on data from a graph. Start by finding the point that represents
the rodent with the fewest litters per year. Since the variable along the x-axis is Litters per Year, the
fewest litters per year is the point closest to the left-hand side, at around 1.2 litters per year. Since
the average litter size is the variable along the y-axis, find the corresponding average litter size when
the litters per year is 1.2 The average litter size is 4. The correct answer is (B).

12. D The question asks which point has the smallest ratio of average litter size to litters per year. Set up
average litter size
a ratio as . Fill in the ratio with data from the graph to find the smallest ratio.
litters per year
Points A and B both have 2 litters per year, but point A has a higher average litter size, giving a
5 2
ratio of . Point B has a ratio of = 1 . Eliminate (A) since the ratio is larger than the ratio in (B).
2 2
11.5
Point C has a ratio of about < 3.3 . This is much higher than the ratio in point B. Eliminate
3.5
4
(C). Point D has a ratio of about = 0.8 . This is lower than the ratio in point B. Eliminate (B).
5
The correct answer is (D).

13. A The question asks for a scatterplot based on given information. Since b is positive, the y-intercept
must be positive; eliminate (B) and (C) because they have y-intercepts that are not positive. The x
in the equation is squared, meaning the graph will be a parabola. The a coefficient on the x 2 term
is negative, which means that the parabola would open down toward the negative y-axis and not
up toward the positive y-axis. Eliminate (D) because it has a parabola that opens up. The correct
answer is (A).

14. B The question is asking for a type of function that is represented by data. Look at the answer choices
to determine the possible options. All answer choices refer to either exponential or linear changes,
and those changes could be increases/growth or decreases/decay. As the time increases in the chart,
the population also increases. Eliminate (A) and (C) which indicate a decreasing population over
time. Now determine if the increase is linear or exponential. A population with linear growth
will increase by the same amount for each unit of time. From a time of 0 minutes to a time of 17
minutes, the population increases from 1 to 2, for an increase of 1. From 17 minutes to 34 minutes,
another 17 minutes of time passes, but the population increases from 2 to 4 for an increase of 2.
Since the population is not increasing by the same amount every 17 minutes, the growth is not
linear. Eliminate (D). Choice (A) indicates exponential growth, which means that the popula-
tion is growing by a multiplier each unit of time. From 0 minutes to 17 minutes, the population
doubles, and it doubles again over the next 17 minutes to the time of 34 minutes. If the population
doubles every 17 minutes, then the population is showing exponential growth. The correct answer
is (B).

3 6   |   Practice Test 6: Answers and Explanations


SAT Premium Prep

15. C The question asks for an expression for a specific scenario based on given information. The ques-

tion states that y is the number of years after 2010. The question also gives that there are 15,000

people on January 1, the initial day, in City X, and the population increases by 1.2% per year.

Therefore, to estimate the population of the city 10 years after 2010, plug y = 10 into the expres-
10
⎛ 1.2 ⎞
sion to get 15, 000 ⎜1 + . Follow the same steps to find the estimated population of the city
⎝ 100 ⎟⎠
5
1.2 ⎞
5 years after 2010 by plugging in y = 5 to get 15, 000 ⎛⎜1 + . To find the difference, subtract
⎝ 100 ⎟⎠
10 5
1.2 ⎞ ⎛ 1.2 ⎞ . The correct answer is (C).
these two expressions to get 15, 000 ⎛⎜1 + − 15, 000 ⎜1 +
⎝ 100 ⎟⎠ ⎝ 100 ⎟⎠
16. A The question asks for the meaning of a part of a given equation. At the y-intercept of an equation,
x = 0. Plug x = 0 into the equation to get y = C + (I + S)(0) = C + 0 = C . This represents the cost of
the computer. The correct answer is (A).

17. C The question asks for the number of months that the total cost will be less using the data for one
business vs. another. Use the equation given to set up an inequality. The cost for Cat’s Computers
will be 990 + (50 + 65)x, and the cost for Bobby’s Bounty will be 1,100 + (60 + 45)x . Because the
cost of Cat’s Computers is less than or equal to the cost of Bobby’s Bounty, make the first expres-
sion less than or equal to the second: 990 + (50 + 65)x ≤ 1,100 + (60 + 45)x. Solve for x. Start by
adding like terms in the parentheses to get 990 + 115x ≤ 1,100 + 105x. Subtract 990 from both
sides of the inequality to get 115x ≤ 110 + 105x. Subtract 105x from both sides of the inequality to
get 10x ≤ 110. Divide both sides by 10 to get x ≤ 11. The correct answer is (C).

18. D The question asks for a representative graph based on a specific scenario. If Norepinephrine loses
half of its pharmacological activity every 2 minutes, then pharmacological activity must be
decreasing as time increases. Eliminate (B). Losing 50% pharmacological activity every 2 minutes
is exponential decay, because the actual amount lost each time period is not constant. Eliminate
(A) and (C) since they do not represent exponential decay. The correct answer is (D).

19. A The question asks for the number of glasses that can be filled from a full bottle of grape juice. Start by
1 2
finding the volume of the glass. The volume of a cone is given by the equation V = r h . Because
3
the diameter is 5 inches, the radius is 2.5 inches. The height is 4 inches. The volume of each glass
1 2
is V =  (2.5) ( 4 ) ≈ 26.18 cubic inches. One bottle contains approximately 46 cubic inches, so
3
divide the number of cubic inches in a bottle of grape juice by the number of cubic inches per glass

to get 46 ÷ 26.18 = 1.76. Because the question wants the glasses to be completely filled, round

down to 1. The correct answer is (A).

Practice Test 6: Answers and Explanations  |   3 7


SAT Premium Prep

20. D The question asks for the greatest possible value of an expression. It is possible to solve the given
inequality for x and then put that into the expression in the question. To save time, though, make
sure to read the final question, which ask about the value of 2x – 3. Solve the inequality to get the
2x term by itself. Subtract 3 from both sides of the inequality to get 2x ≤ 1. Now, to find 2x – 3,
subtract 3 from both side of the inequality to get 2x – 3 ≤ 1 – 3. This simplifies to 2x – 3 ≤ –2.
Therefore, the greatest possible value of the expression is –2. The correct answer is (D).

21. D The question asks for the length of an arc in a circle. Because XZ is a diameter, arc 
XYZ will be

half of the circumference. The radius is 2, so the diameter is double the radius, or 4. Circumfer-

ence has the formula C = πd, so the circumference is 4π. Divide the circumference in half to get

the length of arc 


XYZ , so 4π ÷ 2 = 2π. The correct answer is (D).

22. D The question asks for a percent based on provided data. The words percent greater than or percent less
difference
than indicate percent difference, which is defined as ×100 . Male deaths from colorectal
original
cancer are about 5,000, and female deaths are about 4,200. Because the question asks for percent

greater, the original is the smaller value. The difference is 5,000 – 4,200 = 800, so the percent
800
difference is × 100 = 19% . The correct answer is (D).
4, 200
23. B The question asks for the number of points plotted below a specific line. To find the number of
points, draw the line y = x . Label the horizontal axis as “number of male deaths” and the vertical
axis as “number of female deaths.”
y
y=x
10
number of female deaths

x
2 4 6 8 10
number of male deaths

3 8   |   Practice Test 6: Answers and Explanations


SAT Premium Prep

Points below the line will have an x-value (number of male deaths) that is greater than their y-value
(number of female deaths). Graph the points on the coordinate plane.
y
y=x
10
lung

number of female deaths


8

6
breast
4 colorectal
pancreas
2
prostate
x
2 4 6 8 10
number of male deaths

Lung, colorectal, and prostate cancers have more male deaths than female deaths, so there are 3
points that are below the line. The correct answer is (B).

24. A The question asks for a true statement regarding the results of a study that was conducted. Read
each answer carefully and use process of elimination. All the answer choices refer to standard
deviation. Standard deviation is a measure of the spread of values in a data set. The more spread
out the numbers are from the mean value, the greater the standard deviation is. Ms. Minster’s class
has 16 students with 97%, and there is at least 1 student who scored in each of the different score
categories. This data largely falls into the 97% category, so it would be closely grouped around the
mean. Dr. Chiu’s class has several students with the same score for all scores except 96%, so the
data is more spread out and not clustered around one value as with Ms. Minster’s class. This would
give the scores in Dr. Chui’s class a higher standard deviation than the scores in Ms. Minster’s
class. The correct answer is (A).

25. B The question asks for a true statement based on an inequality. There are variables in the question,
so plug in. Make a = –2 and b = 1. It is unnecessary to consider Roman numeral III, as every
answer choice includes III. Test Roman numeral I. Roman numeral I is false because b can be posi-
tive; eliminate (C) and (D). Roman numeral II is true because a must be negative if a is less than
–a. Eliminate (A). The correct answer is (B).

26. B The question asks for the meaning of a coefficient in context. Start by reading the full question,
which asks for the meaning of the number 2.3788. Then label the parts of the equation with
the information given. In the equation, 2.3788 is multiplied by x. The x-axis is median annual
household income, so x can be labeled as “income.” The y-axis is median home price, so y can be
labeled as “home price.” The equation becomes “home price” = 2.3788(“income”) – 2,895.2. Now

Practice Test 6: Answers and Explanations  |   3 9


SAT Premium Prep

eliminate answer choices that are not consistent with these labels. Choice (A) refers to the change
in home price based on income. The two variables are related by the equation, but it is difficult to
determine the relationship. Keep (A) and check the other answers. Choice (B) also has a relation-
ship between the variables; keep (B), too. Choice (C) refers to the ratio of income to home price,
income
which would be written as . The linear equation does not have this ratio, and rear-
home price
ranging the equation would not result in this ratio being equal to 2.3788. Eliminate (C). Choice
(D) say that the home price increased regardless of…income. Based on the graph and the labels of
the equation, different incomes will be associated with different home prices. Eliminate (D). To
check (A) and (B), plug in some numbers. It is easier to plug in an increase of $1 than of $2.3788,
so check (B) first. If the median annual household income is $10,000, then the median home price
is 2.3788(10,000) – 2,895.2 = $20,892.8. If the median annual income increases by $1 to $10,001,
the median home price is 2.3788(10,001) – 2,895.2 = $20,895.1788. Subtract the two amounts to
find the increase in median home price, or 20,895.1788 – 20,892.8 = $2.3788. Thus, an increase of
$1 in the income does increase the home price by 2.3788. The correct answer is (B).

27. C The question asks for a polynomial that is divisible by a specific factor. There are variables in the
answer choices, so plug in. There are a lot of 2’s in the answer choices, so don’t make x = 2; doing
so might make more than one answer choice work. Make x = 5. Find the values of the functions f
and g when x = 5. This becomes f(5) = 2(5)2 – 8(5) + 6 = 50 – 40 + 6 = 16 and g(5) = (5)3 – 4(5)2
+ 3(5) = 125 – 100 + 15 = 40. When x = 5, x + 4 = 9. Check each answer to see if it is divisible by
9. Choice (A) becomes h(5) = f(5) + g(5) = 16 + 40 = 56. This is not divisible by 9; eliminate (A).
Choice (B) becomes l(5) = f(5) + 2g(5) = 16 + 2(40) = 96. This is not divisible by 9; eliminate (B).
Choice (C) becomes m(5) = 2f(5) + g(5) = 2(16) + 40 = 72. This is divisible by 9; keep (C). Check
the remaining answer choice just in case. Choice (D) becomes n(5) = 2f(5) + 2g(5) = 2(16) + 2(40)
= 112. This is not divisible by 9; eliminate (D). The correct answer is (C).

28. C The question asks which value of c means f(x) = c has 3 real solutions. If f(x) = c has three real solu-
tions, then f(x) = c exactly three times. Use the graph provided and draw horizontal lines for each
answer choice. For (A), the value of c is −2. For f(x) = −2, draw the line y = −2 to see where it inter-
sects the curve. The graph of the two functions has only one point of intersection, so it only has
one solution. Eliminate (A). For (B), at y = 0 (the x-axis) the graph only intersects once. Eliminate
(B). For (C), at y = 1.25, the graph shows three points of intersection. There can be only one correct
answer, so if (C) works, there is no need to try (D). The correct answer is (C).

4 0   |   Practice Test 6: Answers and Explanations


SAT Premium Prep

29. A The question asks for the form of a quadratic which gives the solutions as constants or coefficients.
The correct form is the factored form. Eliminate choices that are not completely factored. Elimi-
nate (C), which is incompletely factored, and (D), which is in standard form y = ax 2 + bx + c. To
choose between (A) and (B), plug in to find which one is equivalent to the given equation. Make
2
x = 2. The original equation becomes 3 (2 + 1) − 27 = 0. This is the target value; circle it. Make
x = 2 in (A) to get 3(2 – 2)(2 + 4) = 0; keep (A). Check the remaining answer choice just in case.
Choice (B) becomes 3(2 – 4)(2 + 2) = −24. Eliminate (B). The correct answer is (A).

30. A The question asks for the average in terms of a variable. There are variables in the answer choices,

so plug in. For averages, use the formula T = AN, in which T is the total, A is the average, and N is
3 + 7 10 5(3) + 4 15 + 4 19
the number of things. Make x = 3. This makes a = = =5, b = = = = 9.5 ,
2 2 2 2 2
6(3) + 1 18 + 1 19
and c = = = = 9.5 . The Total of a, b, and c is 5 + 9.5 + 9.5 = 24, and there are 3
2 2 2
things, so the formula becomes 24 = A(3). Divide both sides of the formula by 3 to get 8 = A. This

is the target value; circle it. Make x = 3 in each answer choice and eliminate any answer which

does not equal 8. Choice (A) becomes 2(3) + 2 = 8. This matches the target, so keep (A) but check
1 1
the remaining choices just in case. Choice (B) becomes 3(3) + = 9 . Eliminate (B). Choice
2 2
(C) becomes 4(3) + 4 = 16. Eliminate (C). Choice (D) becomes 6(3) + 6 = 24. Eliminate (D). The

correct answer is (A).


5
31. , .555, or .556
9
The question asks for the increase in degrees Celsius when the temperature increases by one degree

Fahrenheit. There are variables in the question, so plug in. Choose two numbers for F which have

a difference of 1. If F = 32, the equation is simplified, so use F = 32 and F = 33. If F = 32, then
5 5 5 5 5
C = (32 − 32 ) = (0) = 0 . If F = 33, then C = (33 − 32 ) = (1) = . The difference between
9 9 9 9 9
5 5 5
the two values for C is − 0 = . This is an increase of degrees Celsius. The correct answer is
9 9 9
5
, 0.555, or 0.556.
9
32. 430 The question asks for the number of pages written at the end of a certain number of days. Mei
writes at a rate of 6 pages per day. If Mei writes 6 pages per day for 30 days, she will write 6 × 30
= 180 additional pages. She has already written 250 pages, so she will have 250 + 180 = 430 total
pages at the end of the 30-day period. The correct answer is 430.

Practice Test 6: Answers and Explanations  |   4 1


SAT Premium Prep

33. 1 or .333
3
The question asks for a ratio, which can be expressed as a fraction or a decimal. The frequency

1 k
using the heavier weight is , and the frequency of the harmonic oscillator using the lighter
2 9m

1 k
weight is . The ratio of the frequency using the heavier weight to the frequency using the
2 m
1 k
frequency using heavier weight 2  9m
lighter weight is or 1 k . Simplify this fraction by canceling
frequency using lighter weighht
2 m
k
1 9m
in the numerator and denominator to get . Next, put everything under the same radical
2 k
k m

to get 9m . To divide one fraction by another, flip the fraction in the denominator and multiply
k
m
k m km
it by the fraction in the numerator by it to get × = . Cancel km in the numerator
9m k 9km

1 1 1 1
and the denominator to get = = . The correct answer is .
9 9 3 3
34. 672 The question asks for a measurement and gives conflicting units. When dealing with scale maps or
models, make a proportion, being sure to match up units. The question states that a hobbit of oats
1 pound 105 pounds
weighs 105 pounds. The proportion is = . Cross-multiply to get x = 1,680
16 ounces x ounces
ounces. There are 2.5 imperial bushels of oats in a hobbit, so to find how many ounces are in one
imperial bushel, divide the number of ounces by 2.5 to get 1,680 ÷ 2.5 = 672 ounces per bushel of
oats. The correct answer is 672.

35. 5, 6, or 7

The question asks for one possible integer value of the radius of a circle. The arc of a circle

relates to the circumference in the same proportion as the central angle relates to 360⁰, so
arc central angle
= . To find a possible radius for the circle, make the arc 3, the central
circumference 360°
3 40°
angle 40⁰, and the circumference 2πr to get = . Cross-multiply to get (3)(360) = (40)
2r 360°

4 2   |   Practice Test 6: Answers and Explanations


SAT Premium Prep

(2πr), or 1,080 = 80πr. Divide both sides of the equation by 80π to get 4.3 ≈ r. Because r must be

an integer and 3 was the smallest possible arc, round up to 5. Other possible answers are 6 and 7.

The correct answer is 5, 6, or 7.

36. 180 The question asks for an additional number of girls to be surveyed to make a certain fraction. Set

up a proportion. Ashley has surveyed 100 girls and 210 boys for a total of 310 students. She wants
4
girls to make up of the eventual total. If x is the number of additional girls Ashley surveys,
7
then the total number of girls surveyed will be 100 + x and the total number of students surveyed
4 100 + x
will be 310 + x. The proportion is = . Cross-multiply to get 4(310 + x) = 7(100 + x).
7 310 + x
Distribute on both sides to get 1,240 + 4x = 700 + 7x. Subtract 700 from both sides of the equation

to get 540 + 4x = 7x. Subtract 4x from both sides of the equation to get 540 = 3x. Divide by 3 on

both sides of the equation to get 180 = x. The correct answer is 180.

37. 28 The question asks for the population 20 years after 2015. This means that y, the number of
20
years after 2015, is 20. Plug this into the given equation to get P = 20(r )10 , which simplifies to
P = 20(r)2. To find the value of r, use the growth formula, which is final amount = original amount
(1 ± rate)number of changes. The rate is given as an 18% increase, but it must be expressed as a decimal.
Therefore, r = 1 + 0.18 = 1.18. Plug this into the formula to get P = 20(1.18)2 = 27.848. This is in
millions, and the question asks for the value to the nearest million. The correct answer is 28.

38. 1.18 The question asks for the value of a variable in the previous equation. The equation given is a
variation of the equation for exponential growth: final amount = original amount(1 ± rate)number
of changes
, where rate is expressed as a decimal. The population is expected to increase 18 percent
every ten years, so when y = 10, there is only one change. The rate per decade is 18%, which is
0.18 when expressed as a decimal. Because the population is growing, the rate is added to 1, so
r = 1 + 0.18 = 1.18. The correct answer is 1.18.

Copyright © 2022 by TPR Education IP Holdings, LLC. All rights reserved.

Practice Test 6: Answers and Explanations  |   4 3


SAT Premium Prep

RAW SCORE CONVERSION TABLE SECTION AND TEST SCORES


Writing Writing
Raw Score Math Raw Score Math
Reading and Reading and
(# of correct Section (# of correct Section
answers) Test Score Language answers) Test Score Language
Score Score
Test Score Test Score
0 200 10 10 30 530 28 29
1 200 10 10 31 540 28 30
2 210 10 10 32 550 29 30
3 230 11 10 33 560 29 31
4 240 12 11 34 560 30 32
5 260 13 12 35 570 30 32
6 280 14 13 36 580 31 33
7 290 15 13 37 590 31 34 Please note that the
8 310 15 14 38 600 32 34 numbers in the table may
9 320 16 15 39 600 32 35
10 330 17 16 40 610 33 36
shift slightly depending
11 340 17 16 41 620 33 37 on the SAT’s scale from
12 360 18 17 42 630 34 38 test to test; however, you
13 370 19 18 43 640 35 39
14 380 19 19 44 650 35 40
can still use this table to
15 390 20 19 45 660 36 get an idea of how your
16 410 20 20 46 670 37 performance on the prac-
17 420 21 21 47 670 37
tice tests will translate to
18 430 21 21 48 680 38
19 440 22 22 49 690 38 the actual SAT.
20 450 22 23 50 700 39
21 460 23 23 51 710 40
22 470 23 24 52 730 40
23 480 24 25 53 740
24 480 24 25 54 750
25 490 25 26 55 760
26 500 25 26 56 780
27 510 26 27 57 790
28 520 26 28 58 800
29 520 27 28

CONVERSION EQUATION SECTION AND TEST SCORES


Convert

READING TEST READING TEST
RAW SCORE SCORE
(0–52) (10–40)

Convert
 + = x 10 =
WRITING AND WRITING AND READING TEST READING AND EVIDENCE-BASED READING
LANGUAGE TEST LANGUAGE TEST SCORE WRITING TEST AND WRITING
RAW SCORE SCORE (10–40) SCORE SECTION SCORE
(0–44) (10–40) (20–80) (200–800)

Convert
+ =  + =
MATH TEST MATH TEST MATH SECTION MATH SECTION EVIDENCE-BASED READING TOTAL SAT
NO CALCULATOR CALCULATOR RAW SCORE SCORE AND WRITING SCORE
RAW SCORE RAW SCORE (0–58) (200–800) SECTION SCORE (400–1600)
(0–20) (0–38) (200–800)

4 4   |   Practice Test 6: Answers and Explanations

You might also like